You are on page 1of 103

www.insightsonindia.

com www.insightsias.com
TEST 8 Solutions

Prelims 2017 Test 8


Solutions
1. The International Court of Justice (ICJ) is the principal judicial
organ of the United Nations (UN). Consider the following about it.
1. The ICJ judges are elected for terms of office of nine years by
the United Nations General Assembly and the Security Council.
2. The Court can take suo moto cognizance of disputes between
nation-states.
3. The seat of the Court is in the Hague.
4. ICJ gives advisory opinions on legal questions at the request of
the organs of the United Nations.
Select the correct answer using the codes below.

a) 1 and 3 only
b) 1, 3 and 4 only
c) 2 and 4 only
d) 1, 2, 3 and 4

Solution: b)
Justification: Statement 1: These organs vote simultaneously but
separately. In order to be elected, a candidate must receive an absolute
majority of the votes in both bodies. This sometimes makes it necessary
for a number of rounds of voting to be carried out.

In order to ensure a measure of continuity, one third of the Court is


elected every three years. Judges are eligible for re-election.

Statement 2: The Court is competent to entertain a dispute only if the


States concerned have accepted its jurisdiction in one or more of the
following ways:

by entering into a special agreement to submit the dispute to the


Court;

by virtue of a jurisdictional clause,

through the reciprocal effect of declarations made by them


Statement 4: The International Court of Justice acts as a world court.
The Court has a dual jurisdiction : it decides, in accordance with


INSIGHTS PRELIMS TEST SERIES 2017

www.insightsonindia.com www.insightsias.com
TEST 8 Solutions

international law, disputes of a legal nature that are submitted to it by


States (jurisdiction in contentious cases); and it gives advisory opinions
on legal questions at the request of the organs of the United Nations or
specialized agencies authorized to make such a request (advisory
jurisdiction).

Q Source: Syllabus: United Nations Website

2. Duty credit scrip is one of the most important export promotion


incentive provided by the government at present to exporters.
What is a Duty credit scrip?
a) It is a pass that allows the holder to import commodities by not
paying a specified amount in import duties.
b) It is an explicit subsidy that is granted in proportion to the value
added to products by exporters.
c) It a corporate tax exemption granted to export processing zones.
d) It is subsidized credit that allows exporters to dump products at
a cheaper price abroad.

Solution: a)
Learning: It is one of the important components of the Foreign Trade
Policy, 2015-2020.

The approach of the scheme is to promote exports by giving tax


incentives to the exporters.

Following are the main features of duty credit scrip:

They are issued to exporters


The scrip allows duty deduction (non-payment of taxes) of a
specified amount in the scrip. The scrip value or the amount of tax
deduction will be specified in the scrip.


INSIGHTS PRELIMS TEST SERIES 2017

www.insightsonindia.com www.insightsias.com
TEST 8 Solutions

The scrip value or tax reduction is expressed as a percentage of


export turnover of the exporter.
The scrip value usually varies between 3 per cent to 5 per cent
under Foreign Trade Policy 2015.
Q Source: Current affairs: Foreign trade policy 2015-2020

http://www.indianeconomy.net/splclassroom/16/what-is-duty-credit-
scrip/

3. If there was no Right to constitutional remedies in the


Constitution, which of the following would follow?
a) A citizen would not be able to move court at all.
b) There would be no rule of law in the country.
c) Separation of powers between legislature and executive would
be completely diluted.
d) A democratic government may turn authoritarian.

Solution: d)
Provision: Dr. Ambedkar considered the right to constitutional
remedies as heart and soul of the constitution. It is so because this right
gives a citizen the right to approach a High Court or the Supreme Court
to get any of the fundamental rights restored in case of their violation.

The Supreme Court and the High Courts can issue orders and give
directives to the government for the enforcement of rights.

The courts can issue various special orders known as writs.

Justification: Option A: A citizen will not be able to move court only


for getting his constitutional rights enforced. He can move courts for
other purposes such as resolving a legal dispute. So, A is wrong.

Option B: Rule of law emanates from Article 14 which essentially


considers everyone equal before law. Even if the constitutional remedies
cease to exist, rule of law would continue to operate. Citizens may still be
able to enjoy fundamental rights, but they may not be able to complaint


INSIGHTS PRELIMS TEST SERIES 2017

www.insightsonindia.com www.insightsias.com
TEST 8 Solutions

against its infringement. Moreover, all other legal rights and civil,
criminal laws will be operational. So, B is wrong.

Option C: It is an absurd statement.

Option D: If citizens cannot get their fundamental rights enforced,


governments may very well breach these rights, such as censoring
newspapers, imposing curfews and emergency like situations. So, D is
correct.

Q Source: Page 42: Indian Constitution at Work: NCERT XIth

4. These tribes are recognised among other Naga people by their


tattoos, found all over their face and hands. They are also known as
Indias last surviving headhunters. The tribe is
a) Aangla
b) Nori
c) Konyak
d) Mishmi

Solution: c)
Learning: The Konyak have the largest population among the Nagas.

Their facial tattoos were earned for taking an enemy's head. They are
called the land of Angh's.

Aoleang is the biggest festival for the Konyak, falling in the month of
April.

The Konyaks can be found in Myanmar, in the Tirap and Changlang


districts of Arunachal, and in the Mon district of Nagaland, India. They
are known in Arunachal as Wancho Konyak.

The Konyaks are known for their fierce headhunting history, which
continued until 1960.


INSIGHTS PRELIMS TEST SERIES 2017

www.insightsonindia.com www.insightsias.com
TEST 8 Solutions

http://www.dailymail.co.uk/news/article-2638214/Last-headhunters-
Amazing-images-document-tattoo-adorned-faces-historic-warriors-
fierce-cultural-existence-soon-disappear-forever.html

http://www.bbc.com/travel/story/20140811-indias-last-surviving-
headhunters

Q Source: Major tribes North-eastern India

5. Consider the following about Zero Hunger Challenge.


1. It was launched at the Rio+ conference by the United Nations
Secretary General.
2. It will fund agricultural public stockholding programmes in
developing nations and assist in mitigating adverse effects of the
climate change.
Which of the above is/are correct?


INSIGHTS PRELIMS TEST SERIES 2017

www.insightsonindia.com www.insightsias.com
TEST 8 Solutions

a) 1 only
b) 2 only
c) Both 1 and 2
d) None

Solution: a)
Justification: Statement 1: The Zero Hunger Challenge was launched
by United Nations Secretary-General Ban Ki-moon in 2012. The Zero
Hunger vision reflects five elements from within the SDGs, which taken
together, can end hunger, eliminate all forms of malnutrition, and build
inclusive and sustainable food systems.

Statement 2: It focuses on an End to Rural Poverty, malnutrition;


sustainable food production and consumption.

But, it does not envisage funding of public stockholding programmes as


such. You should be careful to eliminate such statements.

Q Source: Syllabus: United Nations Website

6. Consider the following about the North Atlantic Treaty


Organization (NATO).
1. NATO membership is open to any country that is willing to
accept the terms of the military alliance.
2. All decisions are taken by consensus in NATO.
3. The Membership Action Plan (MAP) of NATO is a military code
that prohibits attack on member countries by any of the NATO
members.
Select the correct answer using the codes below.

a) 1 only
b) 2 and 3 only
c) 2 only
d) None of the above


INSIGHTS PRELIMS TEST SERIES 2017

www.insightsonindia.com www.insightsias.com
TEST 8 Solutions

Solution: c)
Justification: Statement 1: NATO membership is open to any other
European state in a position to further the principles of this Treaty and
to contribute to the security of the North Atlantic area.

Statement 2: A NATO decision is the expression of the collective will of


all 28 member countries since all decisions are taken by consensus.

NATO is committed to the peaceful resolution of disputes. If diplomatic


efforts fail, it has the military capacity needed to undertake crisis-
management operations. These are carried out under the Washington
Treaty - NATOs founding treaty - or under a UN mandate, alone or in
cooperation with other countries and international organizations.

Statement 3: The Membership Action Plan (MAP) is a NATO programme


of advice, assistance and practical support tailored to the individual
needs of countries wishing to join the Alliance.

Q Source: Current Affairs + Improvisation: Map-based questions:


Europe

7. The Twin balance sheet challenge and twin deficit often seen in
news include which of these economic variables?
1. Current Account Deficit
2. Fiscal Deficit
3. External debt
4. Banking profits
Select the correct answer using the codes below.

a) 3 and 4 only
b) 1, 2 and 4 only
c) 1 and 2 only
d) 1, 2, 3 and 4

Solution: b)


INSIGHTS PRELIMS TEST SERIES 2017

www.insightsonindia.com www.insightsias.com
TEST 8 Solutions

Justification: The balance sheets of both public sector banks (PSBs)


and some corporate houses are in very bad shape and it has been seen as
a major obstacle to investment and reviving growth.

The problems faced by the Public Sector Banks are linked directly
to that of the corporate sector. During the boom years, some
companies borrowed a lot of money from banks to invest in
infrastructure and commodity- related businesses, such as steel,
power, infrastructure etc.
But now, due to slump in both these sectors, the corporate profits
have hit new lows. With low profits, the corporates are not able to
repay their loans and their debts are rising at an alarming level.
They have no other option other than to cut back investments.
The high amount of NPAs in banks have hit their profitability as
well, as banks have to make more provisioning. Banks have to set
aside large funds as provisions to take care of the potential losses
arising out of the loans that might go bad. NPAs are the reason why
most banks reported losses in the last quarter.
This is known as the Twin balance sheet challenge. You must be already
aware about the Twin deficit challenge.

Q Source: http://indianexpress.com/article/business/economy/rising-
npa-burden-twin-balance-sheet-challenge-biggest-hurdle-to-recovery/

8. Consider the following matches of famous paintings with the


locations they are found in.
1. Chitharal Jain Monuments : Thirunadhikkara Caves
2. Gajendra Moksham : Krishnapuram Palace, Kayamkulam
3. Anantha Shayanam : Pallikurup Mahavishnu Temple
Select the correct answer using the codes below.

a) 1 and 2 only
b) 1 and 3 only
c) 2 and 3 only
d) 1, 2 and 3


INSIGHTS PRELIMS TEST SERIES 2017

www.insightsonindia.com www.insightsias.com
TEST 8 Solutions

Solution: c)
Justification: Statement 1: The Chitharal Jain Monuments are the
ruins of a Jain training centre from the 9th century CE, on a small hill in
Chitharal village, Kanyakumar.

Thirunandikkara Cave Temple is a rock-cut cave temple of Pallava art


dated to the seventh and eighth century AD.

Statement 2: The Liberation of Gajendra is a Puranic legend from the


8th Skandha of the Bhagavata Purana.

In this episode, Vishnu came down to earth to protect Gajendra, the


elephant, from the clutches of Makara, the Crocodile, and with Vishnu's
help, Gajendra achieved moksha, or salvation.

Statement 3: The Anantha Shayanam mural painting in the Pallikurup


Mahavishnu Temple, Mannarkkad Palakkad District and the mural
paintings in the sanctom of Padmanabha temple at Thiruvananthapuram
are very famous Kerala mural works.

Q Source: Improvisation: Page 104: NCERT XIth: Living Craft


Traditions of India

9. Consider the following about National Agriculture Market (NAM).


1. It is a pan-India electronic trading portal.
2. It has replaced the Agricultural Produce Marketing Committee
(APMC) in all the states of India.
3. NAM price discovery mechanism will replace the Minimum
Support Mechanism (MSP) from the next financial year.
Select the correct answer using the codes below.

a) 1 only
b) 1 and 3 only
c) 2 and 3 only


INSIGHTS PRELIMS TEST SERIES 2017

www.insightsonindia.com www.insightsias.com
TEST 8 Solutions

d) 1, 2 and 3

Solution: a)
Justification: Statement 2: NAM networks the existing APMC mandis
to create a unified national market for agricultural commodities. It has
not replaced them. So, 2 is incorrect.

Statement 3: There is no such provision.

The NAM Portal provides a single window service for all APMC related
information and services. This includes commodity arrivals & prices, buy
& sell trade offers, provision to respond to trade offers, among other
services. While material flow (agriculture produce) continue to happen
through mandis, an online market reduces transaction costs and
information asymmetry.

Learning: Agriculture marketing is administered by the States as per


their agri-marketing regulations, under which, the State is divided into
several market areas, each of which is administered by a separate
Agricultural Produce Marketing Committee (APMC) which imposes its
own marketing regulation (including fees).

This fragmentation of markets, even within the State, hinders free


flow of agri commodities from one market area to another and
multiple handling of agri-produce and multiple levels of mandi
charges ends up escalating the prices for the consumers without
commensurate benefit to the farmer.
NAM addresses these challenges by creating a unified market
through online trading platform, both, at State and National level
and promotes uniformity, streamlining of procedures across the
integrated markets, removes information asymmetry between
buyers and sellers and promotes real time price discovery, based
on actual demand and supply.
It further promotes transparency in auction process, and access to
a nationwide market for the farmer, with prices commensurate
with quality of his produce and online payment and availability of
better quality produce and at more reasonable prices to the
consumer.
Q Source: http://indianexpress.com/article/india/india-news-
india/whats-in-national-agriculture-market/


INSIGHTS PRELIMS TEST SERIES 2017

www.insightsonindia.com www.insightsias.com
TEST 8 Solutions

10. Consider the following statements.


Assertion (A): A Ministry which loses confidence of the Rajya
Sabha is obliged to resign.

Reason (R): The Council of Ministers is collectively responsible to


the Rajya Sabha.

In the context of the above, which of these is correct?

a) A is correct, and R is an appropriate explanation of A.


b) A is correct, but R is not an appropriate explanation of A.
c) A is correct, but R is incorrect.
d) Both A and R are incorrect.

Solution: d)
Justification: One of the most important features of a parliamentary
democracy is that the executive is routinely under the control and
supervision of the legislature.

This provision means that a Ministry which loses confidence of the


Lok Sabha (Rajya Sabha) is obliged to resign. The principle
indicates that the ministry is an executive committee of the
Parliament and it collectively governs on behalf of the Parliament.

Collective responsibility is based on the principle of the solidarity


of the cabinet. It implies that a vote of no confidence even against a
single minister leads to the resignation of the entire Council of
Ministers.

It also indicates that if a minister does not agree with a policy or


decision of the cabinet, he or she must either accept the decision or
resign. It is binding on all ministers to pursue or agree to a policy
for which there is collective responsibility.
Q Source: Page 92: Indian Constitution at Work: NCERT XIth


INSIGHTS PRELIMS TEST SERIES 2017

www.insightsonindia.com www.insightsias.com
TEST 8 Solutions

11. Winter rainfall in Northern India in Punjab and Haryana occurs


mainly due to
a) Arrival of weak temperate cyclones from the Mediterranean Sea
b) Northeast monsoon crossing over the Bay of Bengal
c) Anti cyclonic circulation patterns in Northern India in winter
d) All of the above

Solution: a)
Justification & Learning: Option A: A Western Disturbance is an
extra-tropical storm originating in the Mediterranean region that brings
sudden winter rain to the north-western parts of the Indian
subcontinent. It is a non-monsoonal precipitation pattern driven by the
westerlies. The moisture in these storms usually originates over the
Mediterranean Sea and the Atlantic Ocean

Western Disturbances are important for the development of the Rabi


crop, which includes the locally important staple wheat.

Option B: During October and November, northeast monsoon while


crossing over the Bay of Bengal, picks up moisture and causes torrential
rainfall over the Tamil Nadu coast, southern Andhra Pradesh, southeast
Karnataka and southeast Kerala. But, this is NOT the case in Northern
India.

Option C: Winter monsoons do not generally cause rainfall in India as


they move from land to the sea. It is because firstly, they have little
humidity; and secondly, due to anti cyclonic circulation on land, the
possibility of rainfall from them reduces. Western cyclonic disturbances
are cyclonic and not anti-cyclonic. So, C is incorrect.

Q Source: Page 45: 11th NCERT: India Physical Geography


INSIGHTS PRELIMS TEST SERIES 2017

www.insightsonindia.com www.insightsias.com
TEST 8 Solutions

12.A Council for Trade Development and Promotion was constituted


in 2015 to ensure
1. Administration of export promotion schemes
2. Cooperation between the Centre and the State for export
promotion
Which of the above is/are correct?

a) 1 only
b) 2 only
c) Both 1 and 2
d) None

Solution: b)
Justification: Statement 2: The government has set up a Trade
Facilitation Council with representations from states and Union
territories to mainstream them in the process of international trade and
assist them with their infrastructural needs.

Statement 1: It is not an executive body. It will make the states active


partners in boosting Indias exports. The first meeting of the council was
held in 2016.

The move comes amid declining exports which fell around 6 per cent,
rather than growing.

Q Source: http://indianexpress.com/article/business/business-
others/with-aim-to-promote-exports-govt-to-set-up-trading-facilitation-
council/

13. Consider the following statements.


Assertion (A): The Rivers that originate in Western Ghats flow only
eastwards.


INSIGHTS PRELIMS TEST SERIES 2017

www.insightsonindia.com www.insightsias.com
TEST 8 Solutions

Reason (R): The Indian Peninsula slopes downward to the eastern


side from the Western Ghats.

In the context of the above, which of these is correct?

a) A is correct, and R is an appropriate explanation of A.


b) A is correct, but R is not an appropriate explanation of A.
c) A is incorrect, but R is correct.
d) Both A and R are incorrect.

Solution: c)
Justification: If there is a slope to the east, the same must also be a
slope for west. So, R fails to justify the assertion. Also, the assertion itself
is incorrect.

The Rivers that originate in Western Ghats flow eastwards or


westwards.
The major river systems originating in the Western Ghats include
Godavari, Kaveri, Krishna, Thamiraparani and Tungabhadra.
These rivers flow to the east due to the gradient of the land and
drain out into the Bay of Bengal.
Major tributaries include Bhadra, Bhavani, Bhima, Malaprabha,
Ghataprabha, Hemavathi and Kabini. Periyar, Bharathappuzha,
Netravati, Sharavathi, Mandovi and Zuari rivers flow westwards
towards the Western Ghats, draining into the Arabian Sea and are
fast-moving, owing to the steeper gradient.
Q Source: Chapter 3: 11th NCERT: India Physical Geography

14.Consider the following about Chhau dance.


1. It is a martial arts dance performed exclusively by men.
2. It is a major cultural symbol of the Eastern Himalayan Hills.
3. Bhulya and Hos tribes are associated with this dance form.
Select the correct answer using the codes below.

a) 1 and 2 only


INSIGHTS PRELIMS TEST SERIES 2017

www.insightsonindia.com www.insightsias.com
TEST 8 Solutions

b) 3 only
c) 1 and 3 only
d) 2 and 3 only

Solution: c)
Justification: Chhau is a style performed exclusively by men from the
triangular area where Bihar, Bengal and Orissa meet. This is the tribal
belt of India home to the tribal groups of Bhulya, Santhals, Mundas, Hos
and Oraons.

In the olden days the powerful Ganga and Gajapati rulers of Orissa
extended their territory from the river Ganges in the north to the
Godavari in the south with the help of a vast army of valiant
Paikas.

They were not in the regular payroll of the army, but received huge
land grants from the kings and the chieftains. They formed the
rank of a peasant-militia.

Though agriculture was their main occupation they used to keep


themselves prepared by regular practice and training in war
techniques.
Q Source: Page 114: NCERT XIth: Living Craft Traditions of India

15. Which of the following statements with reference to the Office of


the Vice President of India is correct?
a) State legislatures are not part of the electoral college of the Vice-
President.
b) The Vice President may be removed from his office by a
resolution of the Rajya Sabha passed by a majority and agreed
to by the Lok Sabha.
c) The Vice President acts as the ex-officio Chairman of the Rajya
Sabha and takes over the office of the President when there is a
vacancy.


INSIGHTS PRELIMS TEST SERIES 2017

www.insightsonindia.com www.insightsias.com
TEST 8 Solutions

d) All of the above

Solution: d)
Learning: Option A:The Vice President is elected for five years. His
election method is similar to that of the President, the only difference is
that members of State legislatures are not part of the electoral college.

Option B: The impeachment of Vice President is different from that of


the President. In the latter case, both houses must pass the resolution by
a special majority.

But, the VP may be removed from his office by a resolution of the Rajya
Sabha passed by a majority and agreed to by the Lok Sabha.

Option C: The Vice President acts as the President only until a new
President is elected. B. D. Jatti acted as President on the death of
Fakhruddin Ali Ahmed until a new President was elected.

Q Source: Page 88: Indian Constitution at Work: NCERT XIth

16.Global Corruption Report is published by


a) Transparency International
b) Association for Democratic Reforms
c) Human Rights Campaign
d) World Vision and Oxfam International jointly

Solution: a)
Justification & Learning: Oxfam international works mainly to
eradicate poverty and does not involve itself into purely political issues
So, D is wrong.

ADR is mainly concerned with Indian Politics. So B cant be the answer.


INSIGHTS PRELIMS TEST SERIES 2017

www.insightsonindia.com www.insightsias.com
TEST 8 Solutions

The Global Corruption Report (GCR) is one of Transparency


Internationals flagship publications, bringing the expertise of the anti
corruption movement to bear on a specific corruption issue.

Most recent reports have focused on corruption in climate change, the


private sector, water and the judiciary.

Q Source: International reports: Syllabus

17. Why floods are a frequent occurrence in Brahmaputra river basin?


1. The tributaries of Brahmaputra are large and bring large water
and sediment volume.
2. The Brahmaputra often witnesses heavy rainfall in its
catchment area.
Which of the above is/are correct?

a) 1 only
b) 2 only
c) Both 1 and 2
d) None

Solution: c)
Justification: The Brahmaputra Valley is said to be one of the most
hazard-prone regions of the country according to the National Flood
Commission of India, about 32 lakh hectares or over 40 per cent of the
Assams land is flood-prone due to this.

An important reason apart from the ones mentioned in the question is


the ageing of embankments. Though embankments dont have specific
life-spans, the ones in Assam are designed on the basis of flood data of 15
to 20 years and are supposed to remain fit for 25 to 30 years.


INSIGHTS PRELIMS TEST SERIES 2017

www.insightsonindia.com www.insightsias.com
TEST 8 Solutions

The filling of embankments with sediment, wear and tear and


unscientific distribution across the river valley is one of the major
reasons cited by officials for the flood devastation.

Q Source: Page 26: 11th NCERT: India Physical Geography

18. India received the highest inflow of FDI from which of the
following countries last financial year?
a) Singapore
b) United States of America
c) China
d) Japan

Solution: a)
Learning: The following chart shows the distribution.


INSIGHTS PRELIMS TEST SERIES 2017

www.insightsonindia.com www.insightsias.com
TEST 8 Solutions

It is believed that most of this FDI is round tripped money (scholars have
estimated it to be around 60% from these tax havens).

Moreover, what is noteworthy that historically these nations have not


been the largest FDI sources for India. They have been USA, UK and
Japan historically.

But, with the liberalization of the foreign capital account, companies


based in some of these tax havens like Mauritius have exploited Indian
tax laws and avoided taxes on their investment in India.

Q Source: Often in news

19.Brent Crude is often used as a benchmark for the prices of other


crude oils. It is found in parts of the North Sea off the coast of
a) Finland and Sweden
b) Biscay near France
c) British Isles island in Ireland
d) U.K. and Norway

Solution: d)
Learning: Brent blend makes up more than half of the world's globally
traded supply of crude oil, which is why it makes an obvious choice for
the benchmark of crude oil.


INSIGHTS PRELIMS TEST SERIES 2017

www.insightsonindia.com www.insightsias.com
TEST 8 Solutions

Brent blend is a light and sweet crude oil.

Q Source: Map-based questions: Europe

20. The bureaucracy in India is expected to be politically neutral.


Political neutrality implies that the bureaucracy will not?
1. Take a purely political position on policy matters.
2. Contest for political offices.
3. Implement policies and schemes that have a political objective
Select the correct answer using the codes below.

a) 1 and 2 only
b) 2 only
c) 2 and 3 only
d) 1, 2 and 3

Solution: a)
Justification: Statement 1: Persons working in the State services (Civil
servants) are required to act in the course of their duties in a politically


INSIGHTS PRELIMS TEST SERIES 2017

www.insightsonindia.com www.insightsias.com
TEST 8 Solutions

neutral manner. This includes the requirement to act impartially and to


implement the Government's policies.

The requirement that State servants must be apolitical when carrying out
their duties, functions and powers.

This means, essentially, that State servants must keep their jobs out of
their politics and their politics out of their jobs.

Statement 2: If they contest for such offices, they will no longer be able
to remain neutral in day to day administration and will tend to extract
political gains from their actions.

Statement 3: Most policies have a political objective, for e.g. policy to


distribute natural resources based on auctions, rather than first come
first serve basis, clearly has a political objective. It is to curb the
exploitation of political power by the high and mighty. But, civil servants
must implement these policies, for e.g. by way of executing the auction
sale.

Q Source: Page 95: Indian Constitution at Work: NCERT XIth

21.Dree is an agricultural rite and a fertility festival of the


a) Nyogmas
b) Lahiris
c) Shyoks
d) Apatanis

Solution: d)
Learning: The word Dree is derived from Diiri, which means
purchasing or borrowing of food items when in scarcity or add to the
existing stock in anticipation of lean days.

Dree is named after Diiri Piilo, a month in Apatani calendar.


INSIGHTS PRELIMS TEST SERIES 2017

www.insightsonindia.com www.insightsias.com
TEST 8 Solutions

It involves the sacrifice of fowls, eggs and animals to the gods Tamu,
Metii and Danyi Pilo(Sun and Moon God). The purpose of the festival is
to appease these gods so that famine could be avoided.

You can read more about it here


http://arunachalipr.gov.in/StateFestival_Dree.htm

Q Source: Syllabus: Important tribal festivals

22. Consider the following matches of major physical features of


Europe with relevant regions.
1. Balkan Peninsula : Stretch from Serbia to Bulgaria
2. Rhine River : Origin in Swiss Alps
3. Volga River : Flows through Central Russia
4. Alps Mountains: Passes through France and Germany
Select the correct answer using the codes below.

a) 2 and 3 only
b) 1 and 4 only
c) 3 only
d) 1, 2, 3 and 4

Solution: d)
Justification: Statement 1: It is a major cultural area in Southeast
Europe with different and disputed borders. The region takes its name
from the Balkan Mountains that stretch from the eastern border of
Serbia to the Black Sea at the east of Bulgaria.

The Balkan region was the first area in Europe to experience the arrival
of farming cultures in the Neolithic era.

Statement 2: It begins in the Swiss canton of Graubnden in the south-


eastern Swiss Alps. into the North Sea in the Netherlands. It is crucial for
commercial activities in Europe, especially Germany.


INSIGHTS PRELIMS TEST SERIES 2017

www.insightsonindia.com www.insightsias.com
TEST 8 Solutions

Statement 3: It flows through central Russia and into the Caspian Sea,
and is widely viewed as the national river of Russia.

Eleven of the twenty largest cities of Russia, including the capital,


Moscow, are in the Volga's watershed.

Statement 4: It ranges approximately 1,200 kilometres across eight


Alpine countries: Austria, France, Germany, Italy, Liechtenstein,
Monaco, Slovenia, and Switzerland. The mountains were formed over
tens of millions of years as the African and Eurasian tectonic plates
collided.

The altitude and size of the range affects the climate in Europe

Q Source: Map-based questions: Europe

23. Which of the following is correct about Oceanic Nino Index


(ONI)?
a) It measures the productivity drop in oceans due to El-Nino
events.
b) It shows the frequency of occurrence of Southern Oscilllations
(SOs) in the Indian Ocean.
c) It compares east-central Pacific Ocean surface temperatures to
their long-term average.
d) None of the above

Solution: c)
Justification: The Oceanic Nino Index (ONI) is one of the primary
indices used to monitor the El Nino-Southern Oscillation (ENSO). The
ONI is calculated by averaging sea surface temperature anomalies in an
area of the east-central equatorial Pacific Ocean, in the region (5S to 5N;
170W to 120W). Also, a 3-month time average (running mean) is
calculated in order to better isolate variability closely related to the
ENSO phenomenon.


INSIGHTS PRELIMS TEST SERIES 2017

www.insightsonindia.com www.insightsias.com
TEST 8 Solutions

Learning: From time to time, agricultural production is affected by El


Nio, an abnormal warming of the Pacific waters near Ecuador and
Peru, which disturbs weather patterns around the world.

The 2015 El Nio has been the strongest since 1997, depressing
production over the past year. But if it is followed by a strong La Nia,
there could be a much better harvest in 2016-17.

Q Source: Improvisation: India Physical Environment + Economic


Survey 2015-16

24. The Prime Minister and all the ministers have to be members
of the Parliament. If someone becomes a minister or Prime
Minister without being an MP, which of these follows?
a) He must obtain a special approval from the Parliament to
continue in the office.
b) He should get prior Presidential consent if he continues in the
office for a period of more than six months.
c) Such a person has to get elected to the Parliament within six
months, failing which he stands removed from the office.
d) None of the above

Solution: c)
Learning: Our former PM Manmohan Singh was not a MP when he
was choosen to be as the PM. He was subsequently elected to a Rajya
Sabha seat from Assam within a period of six months. There are many
cases like this.

The essential idea behind this is that in a Paliamentary democracy, none


who is not a part of the Parliament can become a part of the executive.

Q Source: Page 91: Indian Constitution at Work: NCERT XIth


INSIGHTS PRELIMS TEST SERIES 2017

www.insightsonindia.com www.insightsias.com
TEST 8 Solutions

25. Following countries were some of the worst hit countries in


the European sovereign debt crisis. Their correct order of these
from East to West is?
1. Portugal
2. Italy
3. Greece
4. Spain
Select the correct answer using the codes below.

a) 1234
b) 1423
c) 4213
d) 1324

Solution: b)
Justification: In several European countries, private debts arising
from a property bubble (price crashes) were transferred to sovereign
debt (government debt).

This was due to governments bailing out several banks which lost
money in the property bubble. Several bailouts and an
expansionary (lavish) fiscal policy made matters worse.
Several eurozone member states (Greece, Portugal, Ireland, Spain
and Cyprus) were unable to repay or refinance their government
debt or to bail out over-indebted banks.
They had to approach third-parties for debt settlement like other
Eurozone countries, the European Central Bank (ECB), or the
International Monetary Fund (IMF).
The worst hit countries were PIIGS nations Portugal, Ireland,
Italy, Greece and Spain.


INSIGHTS PRELIMS TEST SERIES 2017

www.insightsonindia.com www.insightsias.com
TEST 8 Solutions

Q Source: Map-based questions: Europe

26. Consider the following about Zero Hour in Parliament.


1. MPs are free to raise any matter which they believe to be
important in the Zero Hour.
2. Ministers are bound to reply on the house floor on matters
raised in Zero Hour.
Which of the above is/are correct?

a) 1 only
b) 2 only
c) Both 1 and 2
d) None

Solution: a)
Justification: Statement 1: Apart from deliberating on bills, control
may also be exercised during the general discussions in the House. The
Question Hour, which is held every day during the sessions of


INSIGHTS PRELIMS TEST SERIES 2017

www.insightsonindia.com www.insightsias.com
TEST 8 Solutions

Parliament, where Ministers have to respond to searching questions


raised by the members; Zero Hour where members are free to raise any
matter that they think is important (though the ministers are not bound
to reply), half-an hour discussion on matters of public importance,
adjournment motion etc. are some instruments of exercising control.

Statement 2: Nobody knows which issue a member would raise during


this hour. As a result, questions so raised without prior notice may be of
no importance. Hence, the Ministers are not bound to reply and that too
immediately on the house floor.

Q Source: Page 115: Indian Constitution at Work: NCERT XIth

27. Nothing in this article shall prevent the State from making
any provision for the reservation of appointments or posts in
favour of any backward class of citizens which, in the opinion of
the State, is not adequately represented in the services under the
State. This is a provision found in
a) Directive Principles of State Policy
b) Fundamental Rights
c) Special Constitutional Rights under Part VII
d) National Commission for Backward Classes (NCBC) Act, 1993

Solution: b)
Learning: This is a provision given under Article 16 (4) of the
Constitution.

Article 16 provides for equality of opportunity for all citizens in matters


relating to employment or appointment to any office under the State. It
prohibits discrimination on grounds other than those mentioned in the
article itself.

The NCBC Act, 1993 established the National Commission for Backward
Classes. The commission was the outcome of the direction of the
Supreme Court in the Mandal case judgement.


INSIGHTS PRELIMS TEST SERIES 2017

www.insightsonindia.com www.insightsias.com
TEST 8 Solutions

Q Source: Page 32: Indian Constitution at Work: NCERT XIth

28. The Indian islands of the Arabian sea are


1. Spread from equator zero degree latitude to fifteen degree
latitude north
2. Largely made of coral deposits
Which of the above is/are correct?

a) 1 only
b) 2 only
c) Both 1 and 2
d) None

Solution: b)
Justification: Statement 1: The latitudinal extent of India begins from
nearly 8 degrees north. An Indian island cannot begin from zero degree
latitude. So, 1 is incorrect.

The islands of the Arabian sea include Lakshadweep and Minicoy. These
are scattered between 8N-12N and 71E -74E longitude.

Statement 2: These islands are located at a distance of 280 km-480 km


off the Kerala coast. The entire island group is built of coral deposits.

Minicoy is the largest island.

The entire group of islands is broadly divided by the Eleventh degree


channel, north of which is the Amini Island and to the south of the
Canannore Island.

Q Source: Page 19: 11th NCERT: India Physical Geography


INSIGHTS PRELIMS TEST SERIES 2017

www.insightsonindia.com www.insightsias.com
TEST 8 Solutions

29. Consider the following statements.


1. The institutions run by the state will not give any religious
education to ensure the neutrality of State in matters of religion.
2. The government will not discriminate between educational
institutions, when granting aid, on the basis of their
management being under particular minority communities.
Which of the above is/are correct?

a) 1 only
b) 2 only
c) Both 1 and 2
d) None

Solution: c)
Justification: Statement 1: This is why in state-run schools, religious
texts are not included in the curriculum. Moreover, the dress code and
other symbolic elements are designed in such a way that they do not
reflect any particular religious biases of such schools.

Statement 2: All minorities, religious or linguistic, can set up their own


educational institutions. By doing so, they can preserve and develop
their own culture. The government will not, while granting aid to
educational institutions, discriminate against any educational institution
on the basis that it is under the management of minority community.

But, while granting aid to an educational institution, the government will


not discriminate against any such institution on the basis that it is under
the management of minority community.

Q Source: Indian Constitution at Work: NCERT XIth

30. The Union Government is planning to fast-track several


projects in Indus river basin to increase irrigation area in Jammu


INSIGHTS PRELIMS TEST SERIES 2017

www.insightsonindia.com www.insightsias.com
TEST 8 Solutions

and Kashmir (J&K). Consider the following matches of these


projects with the region they are related with.
1. Prakachik Khows Canal : Kargil
2. Tral Irrigation Project : Pulwama
3. Ravi Canal : Sambha
Select the correct answer using the codes below.

a) 1 and 2 only
b) 2 and 3 only
c) 1 and 3 only
d) 1, 2 and 3

Solution: d)
Justification: These projects will irrigate nearly 2 lakh acres of land.

This decision was taken after India had decided to exploit to the
maximum the water of Pakistan-controlled rivers Indus, Jhelum
and Chenab as per Indus Water Treaty (IWT).
Three projects Prakachik Khows Canal in Kargil, Tral Irrigation
Project in Pulwama and restoration and modernisation of main
Ravi Canal in Jammus Sambha and Kathua are expected to be
completed by this fiscal.
The fourth project of Rajpora Lift Irrigation is planned will be
completed by December 2019.
The funds for these projects will be raised by National Bank for
Agriculture and Rural Development (NABARD).
Q Source: http://www.hindustantimes.com/india-news/govt-decides-
to-fast-track-irrigation-projects-in-indus-basin/story-
veUrap7NSYcQTJTFKNeIML.html

31. Consider the following about Zonal Councils.


1. They are Constitutional bodies setup under Schedule VII of the
Constitution.


INSIGHTS PRELIMS TEST SERIES 2017

www.insightsonindia.com www.insightsias.com
TEST 8 Solutions

2. The north eastern states are not included in these zonal


Councils.
Which of the above is/are correct?

a) 1 only
b) 2 only
c) Both 1 and 2
d) None

Solution: b)
Justification: Statement 1: They were set up under the States
Reorganization Act, 1956. So, they are statutory bodies, not
constitutional bodies.

Statement 2: They are setup to foster Inter-State co-operation and co-


ordination among the States. Currently, there are total five zonal
councils viz. Northern, Western, Eastern, Central and Southern.

The north eastern states are not included in these five zonal Councils.
Their special problems are looked after by North Eastern Council, set up
under the North Eastern Council Act, 1972. It is also headed by Union
Home Minister. The North Eastern Council includes Assam, Arunachal
Pradesh, Manipur, Tripura, Mizoram, Meghalaya and Nagaland. Sikkim
was included in the North Eastern Council in 2002 and excluded from
the eastern council.

Learning: The Zonal Councils are mandated to discuss and make


recommendations on any matter of common interest in field of social
and economic planning, linguistic minorities, border disputes or inter-
State transport etc.

Thus, Zonal Councils are regional fora of cooperative endeavor for States
linked with each other economically, culturally and politically.

Q Source: Frequently in news


INSIGHTS PRELIMS TEST SERIES 2017

www.insightsonindia.com www.insightsias.com
TEST 8 Solutions

32. Of the cyclones hitting the coastal regions of the country, on


an average four out of five hit the eastern shores of peninsular
India. Why is the eastern coast and Bay of Bengal more prone to
tropical cyclones than the Western Coast and Arabian Sea?
a) Breakaway typhoons over the Northwest Pacific move across the
South China Sea into the Bay of Bengal intensifying the
cyclones.
b) Arabian Sea is colder than the Bay of Bengal
c) Both (a) and (b)
d) None of the above

Solution: c)
Justification & Learning: According to the India Meteorological
Department, this is because in addition to the storms that originate in
the southeast Bay of Bengal and the adjoining Andaman Sea, breakaway
typhoons over the Northwest Pacific move across the South China Sea
into the Bay of Bengal, intensifying into cyclones.

As the frequency of typhoons over the Northwest Pacific is about 35% of


the global annual average, the Bay of Bengal is affected.

In contrast, Arabian Sea cyclones are mostly their own formations and
they also generally move north-west, away from Indias west coast.

Besides, the Arabian Sea is colder than the Bay of Bengal, which inhibits
the formation and intensification of the cyclonic system in the former.
Warm sea surface temperature is an ideal platform for cyclones.

Q Source: Page 86: 11th NCERT: India Physical Geography

33. Consider the following about UNESCOs Man and Biosphere


(MAB) Programme.
1. It is an intergovernmental scientific programme that aims to
establish a scientific basis for the improvement of relationships
between people and their environments.


INSIGHTS PRELIMS TEST SERIES 2017

www.insightsonindia.com www.insightsias.com
TEST 8 Solutions

2. It promotes greater involvement of science and scientists in


policy development concerning the wise use of biological
diversity.
3. The latest 2016 World Congress of Biosphere Reserves took
place in Bali.
Select the correct answer using the codes below.

a) 1 and 2 only
b) 3 only
c) 1 and 3 only
d) 1, 2 and 3

Solution: a)
Justification: Statement 1: The Man and the Biosphere Program
(MAB) was launched in 1970. A UNESCO Biosphere Conference in 1968
had brought together representatives of government and non-
governmental organizations to consider what should be done about
threats to the biosphere that were being increasingly perceived by U.N.
Member States.

After this, an International Co-ordinating Council was formed, which


recommended convening panels of experts from the member states. The
ICC also emphasized that establishment of reserves was important to
meet scientific, educational, cultural and recreational needs. This came
to be known as the biosphere reserve project.

Statement 2: The Man and the Biosphere Program (MAB) is one of the
major scientific programs of the UNESCO.

Today, MAB is a set of related scientific research projects with three


focuses:

Minimizing the loss of biological diversity;


Making people aware of how cultural diversity and biological
diversity affect each other; and
Promoting environmental sustainability through the World
Network of Biosphere Reserves.
MAB has a number of other projects, such as the Great Apes Survival
Project, which supports the protection of gorillas and other large primate


INSIGHTS PRELIMS TEST SERIES 2017

www.insightsonindia.com www.insightsias.com
TEST 8 Solutions

species, and the Young Scientists Award, a scholarship aimed primarily


at youth from developing countries.

Statement 3: It took place in Lima, Peru, in March 2016. This will be the
4th World Congress of Biosphere Reserves and it shall develop a new
vision for the decade 20162025.

Q Source: Improvisation: Page 63: 11th NCERT: India Physical


Geography

34. Consider the following matches of the Indian Navy vessels


with what they are known for.
1. Aryaman and Atulya: Navy Fast Patrol Vessels
2. Tihayu : Nuclear Submarine
3. INS Arihant : Surface Torpedo launcher
4. INS Vikramaditya : Aircraft Carrier
Select the correct answer using the codes below.

a) 1 and 4 only
b) 2 and 3 only
c) 1, 2 and 4 only
d) 1, 2, 3 and 4

Solution: a)
Justification: Statement 1 and 3: The Indian Coast Guard Ships
(ICGSs) Aryaman and Atulya were commissioned into Indian Coast
Guard recently. These ships are the eighteenth and nineteenth in the
series of twenty Fast Patrol Vessels (FPVs).

These FPVs are ideal platform for undertaking multifarious close-coast


missions such as surveillance, interdiction and search and rescue.

Statement 2: Indian Navy has commissioned INS Tihayu, a Car Nicobar


Class water jet fast attack craft (WJFAC).


INSIGHTS PRELIMS TEST SERIES 2017

www.insightsonindia.com www.insightsias.com
TEST 8 Solutions

Statement 3: It is the lead ship of India's Arihant class of nuclear-


powered ballistic missile submarines. INS Arihant is to be the first of the
expected five in the class of submarines designed and constructed as a
part of the Indian Navy's secretive Advanced Technology Vessel (ATV)
project.

Statement 4: INS Vikramaditya is the newest and largest ship aircraft


carrier to join Indian Navy. It was made with Russian support.

Learning: Indian Coast Guard (ICG) ICG is Indias statutory multi-


mission paramilitary organization that protects countrys maritime
interests and enforces maritime law.

It was established in 1978 by the Coast Guard Act, 1978 and


operates under the aegis of Union Ministry of Defence.
ICG has jurisdiction over the territorial waters of India including
its contiguous zone and exclusive economic zone (EEZ).
Q Source: Current Affairs: Developments in the Indian Navy

35. Agri-Tech Infrastructure Fund (ATIF) has been recently


established by the government to promote
a) Micro Irrigation in rural areas
b) National Agriculture Market
c) Seed village concept
d) Development of warehousing and storage near Gram
Panchayats

Solution: b)
Learning: Following successive Budget announcements of 2014 and
2015 on setting up an "Agri-Tech Infrastructure Fund" and on National
Market respectively, Department of Agriculture & Cooperation (DAC)
formulated the scheme for Promotion of National Agriculture Market
through Agri-Tech Infrastructure Fund (ATIF).


INSIGHTS PRELIMS TEST SERIES 2017

www.insightsonindia.com www.insightsias.com
TEST 8 Solutions

Integration of agri-markets across the country through the e-


platform is seen as an important measure for overcoming
challenges posed by the present agri-marketing system namely -
fragmentation of State into multiple market areas each
administered by separate APMC, multiple levy of mandi fees,
requirement for multiple license for trading in different APMCs.
Other issues are licensing barriers leading to conditions of
monopoly, poor quality of infrastructure and low use of
technology, information asymmetry, opaque process for price
discovery, high level of market charges, movement controls, etc.
The need to unify markets both at State and National level is,
therefore, clearly the requirement of time, in order to provide
better price to farmers, improve supply chain, reduce wastages and
create a unified national market through provision of the common
e-platform.
Q Source: http://pib.nic.in/newsite/PrintRelease.aspx?relid=137361

36. Recently a wide divergence has been seen in the Wholesale


price Index (WPI) and Consumer Price Index (CPI) inflation
indexes largely due to their varying compositions. Which of these
broad groups have the largest weightage in WPI and CPI
respectively?
a) Manufactured products and food articles
b) Fuel and power; and Core industries
c) Primary articles and Consumer Services
d) All food articles and manufactured products

Solution: a)
Learning: The widening gap in the inflation based on CPI and WPI
series is often a cause for concern for users not conversant with the
scope, structure and purpose of these indices.


INSIGHTS PRELIMS TEST SERIES 2017

www.insightsonindia.com www.insightsias.com
TEST 8 Solutions

While WPI-based inflation continues to be in the negative zone recently,


CPI-based inflation averaged 4.8 per cent during the same period.

Reasons: The WPI series mainly tracks the movement of producer


and bulk transaction prices and its weights are based on the value
of output in different sectors of the economy.
The series is akin to producer price indices compiled in other
countries.
The CPI basket is based on consumer expenditure estimates and
tracks inflation at retail level or the prices consumers pay.
The base years of the two series are eight years apart as WPI base
revision is long overdue. The weighting diagrams of the two series
vary significantly (as shown below).
For e.g. the weight of total food items in the WPI is 24.3 per cent as
compared to 45.9 per cent in the CPI series. The weakness in global
commodity prices, in particular crude prices, during the last one
and half years has been the cause of decline in the WPI.
Tradable commodities have 55 per cent weight and fuel and power
products about 15 per cent in the basket. On the other hand, the
CPI basket consists of commodities as well as services like health
and education which are not included in the WPI basket.
The CPI has negligible crude price pass through owing to negligible
weight of petroleum products.


INSIGHTS PRELIMS TEST SERIES 2017

www.insightsonindia.com www.insightsias.com
TEST 8 Solutions

Q Source: Economic Survey 2015-16

37. These tribes were largely brought to India as slaves by Arabs


around 7th Century. Originally known as Habshish, these tribes
living in Karnataka are descendants of Bantu people of East Africa.
They are
a) Siddis
b) Mophlas
c) Bhil
d) Chechu

Solution: a)
Learning: Siddis are mostly confined to small pockets of villages in the
Indian states of Karnataka, Maharashtra and Gujarat, and the city of
Hyderabad (theres also a sizable population in Pakistan).

They were brought as slaves to India by Arabs as early as the 7th


Century, followed by the Portuguese and the British later on.

Others were free people who came to India as merchants, sailors


and mercenaries before the Portuguese slave trade went into
overdrive.

When slavery was abolished in the 18th and 19th Centuries, Siddis
fled into the countrys thick jungles, fearing recapture and torture.


INSIGHTS PRELIMS TEST SERIES 2017

www.insightsonindia.com www.insightsias.com
TEST 8 Solutions

These African slaves were originally known as Habshis, which is


Persian for Abyssinian (the former name of Ethiopia was
Abyssinia). But those who rose through the ranks of royal retinue
were honoured with the title Siddi.
Q Source: Current Affairs: Important tribes:
http://www.bbc.com/travel/story/20160801-indias-forgotten-jungle-
dwellers

38. The Directive Principles lists in Part IV of the Constitution


essentially imply which of the following?
1. The goals and objectives that we as a society should adopt
2. Certain rights that individuals should enjoy apart from the
Fundamental Rights
3. Some policies that the government should adopt to further
citizen welfare
Select the correct answer using the codes below.

a) 1 and 2 only
b) 2 and 3 only
c) 3 only
d) 1, 2 and 3

Solution: d)
Justification: We get some idea of the vision of makers of our
Constitution by looking at some of the Directive Principles

The governments from time to time tried to give effect to some Directive
Principles of State Policy. They passed several zamindari abolition bills,
nationalized banks, enacted numerous factory laws, fixed minimum
wages, cottage and small industries were promoted and provisions for
reservation for the uplift of the scheduled castes and scheduled tribes
were made.

Statement 1: All these clearly reflect the social goals that we must follow
like equity and justice. So, Statement 1 is correct.


INSIGHTS PRELIMS TEST SERIES 2017

www.insightsonindia.com www.insightsias.com
TEST 8 Solutions

Statement 2: Rights such as equal pay for equal work, child nutrition,
old age security are some of the rights that DPSP mandates the
government to implement. So, 2 is correct.

Statement 3: Statement 3 logically follows from Statement 1 and 2.

Q Source: Indian Constitution at Work: NCERT XIth

39. What do you understand by the term superbugs?


a) An abnormally large insect species that is detrimental for
agriculture
b) A virus that has the ability to replicate without a host
c) A deadly bacteria which cannot be treated by current antibiotics
d) A strain that is ahead than all planetary species on the course of
natural evolution

Solution: c)
Learning: These superbugs increase in number due to persistent use of
anti-biotics. We have covered anti-biotic resistance topic before.

According to recent British study, Superbugs could kill up to 10


million people globally by 2050.
Scientists from University of Sydney (Australia) have found that
mothers milk from marsupials (also known as Tasmanian devils)
could help to deadly superbugs which resist antibiotics.
It is largest carnivorous marsupial in the world. It has stocky and
muscular build, black fur, pungent odour, keen sense of smell and
extremely loud and disturbing screech.
They have found that peptides in the marsupials milk killed
resistant bacteria.
Scientist are hopeful marsupial peptides could eventually be used
to develop new antibiotics for humans to aid the battle against
superbugs.
Q Source: Current Affairs: http://www.livescience.com/56583-
tasmanian-devil-milk-fights-superbugs.html


INSIGHTS PRELIMS TEST SERIES 2017

www.insightsonindia.com www.insightsias.com
TEST 8 Solutions

40. The Union Government has recently sanctioned a Project of


Ro Pax Ferry service. This service is related with
a) Sagarmala project
b) Digital India
c) MUDRA Yojana
d) Start-up India

Solution: a)
Justification: The Union Ministry of Shipping has recently sanctioned
the Capital Dredging Project of Ro Pax Ferry Services between Gogha &
Dahej, in Gulf of Cambay in Gujarat under Sagarmala programme.

Of the total project cost 50% will be funded by the Union


Government under the Sagarmala programme.
This is first of its kind project in India as it will be executed in the
area of worlds 2nd highest tidal range.
Learning: This project on completion will result in reduction in
motorable distance of 231 kms between Gogha and Dahej in Gujarat to
mere 31 kms.

It will pave way for launching Indias first roll on-roll off (Ro-Ro)
vessel passenger ferry services on this route by 2017.
Besides, it will reduce the travel time to nearly 1 hour.
It will also result in savings in fuel, reduction in CO2 emission and
reduction in road congestion.
The project will open up new avenues in coastal shipping &
tourism and help in socio-economic development of proximate
areas. It will also help in utilisation of inland waterways through
River Narmada for shipping goods from industries located
upstream.


INSIGHTS PRELIMS TEST SERIES 2017

www.insightsonindia.com www.insightsias.com
TEST 8 Solutions

Q Source:
http://www.thehindubusinessline.com/economy/logistics/indias-first-
roro-service-likely-to-begin-in-gujarat-next-year/article9234589.ece

41.The Single Transferable Vote system (STV) in India is followed for


the elections of
1. President
2. Rajya Sabha MPs
3. Members of Legislative Council (MLCs)
Select the correct answer using the codes below.


INSIGHTS PRELIMS TEST SERIES 2017

www.insightsonindia.com www.insightsias.com
TEST 8 Solutions

a) 1 and 2 only
b) 3 only
c) 1 only
d) 1, 2 and 3

Solution: a)
Justification: Statement 2: STV is followed for Rajya Sabha elections.
Every State has a specific quota of seats in the Rajya Sabha. The
members are elected by the respective State legislative assemblies. The
voters are the MLAs in that State.

Every voter is required to rank candidates according to her or his


preference. To be declared the winner, a candidate must secure a
minimum quota of votes, which is determined by a formula.

If after the counting of all first preference votes, required number


of candidates fail to fulfil the quota, the candidate who secured the
lowest votes of first preference is eliminated and his/her votes are
transferred to those who are mentioned as second preference on
those ballot papers. This process continues till the required
number of candidates are declared elected
Statement 3: MLCs are chosen in the following manner:

One-third are elected by members of local bodies such as


Corporations, Municipalities, Gram Sabhas, Gram Panchayats,
Panchayat Samitis and Zila Parishads.

One-third are elected by members of Legislative Assemblies of the


State from among the persons who are not members of the
Assembly.

One-twelfth are elected by persons who are graduates of three


years' standing residing in that state.

One-twelfth are elected by persons engaged for at least three years


in teaching in educational institutions within the state not lower
than secondary schools, including colleges and universities.

One-sixth are nominated by the Governor from persons having


knowledge or practical experience in fields such as literature,
science, arts, the co-operativemovement and social service.


INSIGHTS PRELIMS TEST SERIES 2017

www.insightsonindia.com www.insightsias.com
TEST 8 Solutions

Q Source: Chapter 3: Indian Constitution at Work: NCERT XIth

42. The Zero Defect, Zero Effect (ZED) scheme is related to


a) Micro, Small and Medium Enterprises (MSMEs)
b) Nano-technology mission
c) Road construction in densely forested areas
d) Tackling corruption in public procurement

Solution: a)
Justification: The National SC/ST hub and the Zero Defect, Zero Effect
(ZED) scheme for Micro, Small and Medium Enterprises (MSMEs) were
launched recently.

ZED Scheme aims to rate and handhold all MSMEs to deliver top
quality products using clean technology. It will have sector-specific
parameters for each industry.
The slogan of Zero Defect, Zero Effect (ZED) was first mentioned
by PM in his Independence Day speech in 2014.
It was given for producing high quality manufacturing products
with a minimal negative impact on environment.
MSME sector drives almost 38% of nations GDP and around
employs 110 million employees. The scheme will be cornerstone of
the Central Governments flagship Make in India programme,
which is aimed at turning India into a global manufacturing hub,
generating jobs, boosting growth and increase incomes.
Learning: The objective of the SC/ST (Schedule Castes/Schedule
Tribes) Hub is to provide professional support to entrepreneurs from the
SC/ST. It also seeks to promote enterprise culture and entrepreneurship
among the SC/ST population and to enable them to participate more
effectively in public procurement. It will work towards strengthening
market access/linkage, capacity building, monitoring, sharing industry-
best practices and leveraging financial support schemes.


INSIGHTS PRELIMS TEST SERIES 2017

www.insightsonindia.com www.insightsias.com
TEST 8 Solutions

Public Procurement Policy, 2012 specifies that 4% of procurement done


by ministries, departments and CPSEs should be done from enterprises
owned by SC/ST entrepreneurs.

Q Source: http://pib.nic.in/newsite/PrintRelease.aspx?relid=151762

43. Consider the following about the famous Delhi Ridge.


1. It is a northern extension of the Aravalli Range.
2. It protects Delhi from the hot winds of the deserts of Rajasthan.
3. It forms a crucial water divide in Northern India.
Select the correct answer using the codes below.

a) 1 and 2 only
b) 2 only
c) 3 only
d) 1, 2 and 3

Solution: d)
Justification: The Delhi Ridge is said to be the green lungs for the city
and protects Delhi from the hot winds of the deserts of Rajasthan to the
west. The ridge consists of quartzite rocks covering a distance of
approximately 35 km.

It is also responsible for earning Delhi the tag of the world's second
most bird-rich capital city, after Kenya's Nairobi.

It is believed that the dismemberment of the Himalayan river


system was probably due to the Pleistocene upheaval in the
western Himalayas, including the uplift of the Potwar Plateau
(Delhi Ridge), which acted as the water divide between the Indus
and Ganga drainage systems. It continues to be a major water
divide.


INSIGHTS PRELIMS TEST SERIES 2017

www.insightsonindia.com www.insightsias.com
TEST 8 Solutions

Q Source: Improvisation: Page 8: 11th NCERT: India Physical


Geography

44. India has recently successfully completed its nuclear triad.


What do you understand by a Nuclear Triad?
a) Development of reactors that can process uranium, thorium and
plutonium
b) Development of nuclear enrichment technology for small,
medium scale as well as large nuclear plants
c) Association with major nuclear export control regimes
d) None of the above

Solution: d)


INSIGHTS PRELIMS TEST SERIES 2017

www.insightsonindia.com www.insightsias.com
TEST 8 Solutions

Learning: By inducting the first indigenously built strategic nuclear


submarine INS Arihant into service, India becomes sixth country to have
a nuclear triad i.e. capable of delivering nuclear weapons by aircraft,
ballistic missiles and submarine launched missiles.

Other countries having nuclear triad are Russia, United States, United
Kingdom, France and China.

It includes Agni series, Prithvi series, Prahar Missile, Shaurya Missile,


supersonic Brahmos and subsonic Nirbhay missiles. Indian Airforce
(IAF) operates Jaguars aircrafts which are designed for deep penetration
strike and can carry nuclear bomb.

Besides, IAF also has SU 30 MKI and Rafale aircrafts which can also be
used to deliver nuclear weapons.

Q Source: http://www.thehindu.com/news/national/now-india-has-a-
nuclear-triad/article9231307.ece

45. Consider the following statements.


1. Parliamentary privileges are enshrined in the Constitution and
enjoyed by individual MPs as well as each house of Parliament.
2. The presiding officer of the legislature has the final powers in
deciding matters of breach of privilege.
Which of the above is/are correct?

a) 1 only
b) 2 only
c) Both 1 and 2
d) None

Solution: c)
Justification: Statement 1: Privileges may be classified into two
groups:


INSIGHTS PRELIMS TEST SERIES 2017

www.insightsonindia.com www.insightsias.com
TEST 8 Solutions

Privileges that are enjoyed by the members individually and

Privileges that belong to each House (Lok Sabha or Rajya Sabha) of


Parliament collectively.
Parliament has not yet codified its privileges. In fact, the privileges rest
on conventions and they may be ascertained by the practice and law that
is in force In England

Statement 2: The Privileges and immunities enjoyed by the members


individually are:

Freedom of speech - The basic idea of extending this freedom


being the necessity that every member would put forward without
fear or favour his/her arguments for or against any matter before
the House.

Freedom from arrest - From this freedom it is understood that no


such member shall be arrested in a civil case 40 days before and
after the adjournment of the House (Lok Sabha or Rajya Sabha)
and also when the House is in session.

Exemption from attendance as jurors and witnesses.


We will cover collective privileges in later tests.

Q Source: Page 115: Indian Constitution at Work: NCERT XIth

46. Consider the following about the historic Kigali Amendment,


2016.
1. It aims to phase out Chlorofluorocarbons (CFCs) completely by
2030.
2. It puts a legally binding voluntary Carbon dioxide emission
reduction targets on COP member states.
Which of the above is/are correct?

a) 1 only
b) 2 only


INSIGHTS PRELIMS TEST SERIES 2017

www.insightsonindia.com www.insightsias.com
TEST 8 Solutions

c) Both 1 and 2
d) None

Solution: d)
Justification: None of 1 or 2 is correct.

The Kigali Amendment amends the 1987 Montreal Protocol that was
designed to close growing ozone hole in by banning ozone-depleting
coolants like chlorofluorocarbons (CFCs). Thus, amended Montreal
Protocol which was initially conceived only to plug gases that were
destroying the ozone layer now includes HFCs responsible for global
warming.

Under Kigali Amendment, in all 197 countries, including India


have agreed to a timeline to reduce the use of HFCs by roughly
85% of their baselines by 2045.
Developed and developing countries have agreed to different
timelines for each group.
It is different from the Paris agreement.
HFCs are a family of greenhouse gases (GHGs) that are largely
used in refrigerants in home, car air-conditioners and air sprays
etc.
These factory-made gases had replaced CFCs under the 1987
Montreal Protocol to protect Earths fragile protective Ozone layer
and heal the ozone hole over the Antartica.
In recent times, it was found that HFCs have several thousand
times capacity in retaining heat in the atmosphere compared to
carbon dioxide (CO2), a potent GHG.
Currently, HFCs are currently the worlds fastest GHGs, with
emissions increasing by up to 10% each year.
Q Source: http://www.cop22.ma/en/landmark-agreement-reached-
kigali-fight-global-warming


INSIGHTS PRELIMS TEST SERIES 2017

www.insightsonindia.com www.insightsias.com
TEST 8 Solutions

47. Consider the following statements about Japanese


Encephalitis (JE).
1. JE primarily affects children.
2. Individuals cannot develop natural immunity against JE.
Which of the above is/are correct?

a) 1 only
b) 2 only
c) Both 1 and 2
d) None

Solution: a)
Justification: Statement 1: Japanese encephalitis (JE) is a mosquito-
borne (often found in rice fields) flavivirus. It belongs to the same genus
as dengue, yellow fever and West Nile viruses. JE primarily affects
children.

Statement 2: Most adults in endemic countries have natural immunity


after childhood infection, but individuals of any age may be affected.
Major JE outbreaks occur every 2-15 years

Recently it has claimed 54 lives in Odishas backward Malkangiri district


in just more than a month.

JE transmission mainly intensifies during the rainy season, during which


vector populations increase.

There is no specific therapy for JE. Intensive supportive therapy is


indicated.

Q Source: http://www.thehindu.com/news/national/other-
states/malkangiri-hospital-stretched-as-japanese-encephalitis-death-
toll-climbs/article9221417.ece


INSIGHTS PRELIMS TEST SERIES 2017

www.insightsonindia.com www.insightsias.com
TEST 8 Solutions

48. Which of the following differentiates between a private bill


and a public bill?
a) A private bill is not disclosed in public as it contains sensitive
matters, whereas a public bill is published in the Gazette of
India.
b) A private bill is related to non-public goods and commercial
aspects of government administration, whereas public bill
relates to substantial public goods.
c) A private bill can be proposed only to the Council of Ministers,
whereas a public bill is a draft of the proposed law in
Parliament.
d) A private bill is introduced by a non-Minister, whereas a public
bill is introduced by a Minister.

Solution: d)
Learning: A Member of Parliament, other than a Minister (i.e., not a
member of the Government) is known as a Private Member. A Bill
initiated by any such member is called a Private Member's Bill.

A bill proposed by a minister is described as Government Bill. There are


certain restrictions on the introduction of a private members bill.

In Rajya Sabha and Lok Sabha, as also in other State Legislatures, a


particular day of every week is kept aside for transation of Private
Members' Legislative Business. In Rajya Sabha two and half hours i.e.,
from 2.30 p.m. to 5.00 p.m. on every alternate Friday are generally
allotted for transaction of Private Members' Legislative Business.

Q Source: Page 112: Indian Constitution at Work: NCERT XIth

49. These tribes inhabit a tranquil pine clad valley called Ziro in
the Lower Subansiri District of Arunachal Pradesh. They are
famous for their unique practice of wet rice cultivation. They are?
a) Galong
b) Mog
c) Halam


INSIGHTS PRELIMS TEST SERIES 2017

www.insightsonindia.com www.insightsias.com
TEST 8 Solutions

d) None of the above

Solution: d)
Learning: It is difficult to understand how the early Apatanis had
brilliantly discovered the magnificent irrigated rice cultivation without
help of scientific technologies.

Their wet rice cultivation system and their agriculture system are
extensive even without the use of any farm animals or machines. So is
their sustainable social forestry system.

UNESCO has proposed the Apatani valley for inclusion as a World


Heritage Site.

This has done due to its "extremely high productivity" and "unique" way
of preserving the ecology.

Q Source: Current Affairs: Important tribes:


http://www.downtoearth.org.in/news/vale-of-apatanis-56032

http://www.downtoearth.org.in/news/unravelling-the-genetic-secrets-
of-the-symbiodinium-algae-54452

50. That a Money Bill shall not be introduced in the Council of


States is a
a) Parliamentary Convention
b) Parliamentary Legislation
c) Constitutional provision
d) Executive fiat

Solution: c)
Learning: Ordinary bills can be introduced in both the houses and
must be passed by both for enactment.

But, a money bill (we will cover definition when we cover Laxmikanth)
can be introduced only in Lok Sabha.


INSIGHTS PRELIMS TEST SERIES 2017

www.insightsonindia.com www.insightsias.com
TEST 8 Solutions

Once the bill is referred by the Lok Sabha to the Rajya Sabha, RS can
either approve the bill or suggest changes but cannot reject it.

If it takes no action within 14 days the bill is deemed to have been


passed.

Amendments to the bill, suggested by Rajya Sabha, may or may not be


accepted by the Lok Sabha.

Q Source: Page 113: Indian Constitution at Work: NCERT XIth

51. The driving force behind the South-west Monsoons in India is


1. Difference between the sea-land temperature
2. Western disturbances
Which of the above is/are correct?

a) 1 only
b) 2 only
c) Both 1 and 2
d) None

Solution: a)
Justification: Statement 1: Trade winds push moisture laden winds
from a high pressure zone over seas to the low pressure zone over land.
These winds travel to the interior part of the sub-continent and cause
heavy rainfall.

Statement 2: They withdraw in the Monsoon season and ae visible


across Northern India only in the Winter season, when they bring light
rainfall in the states of Punjab and Haryana.

There are other modern theories of Monsoon formation as well, but they
are outside our present scope of a generalist UPSC syllabus.

Q Source: Page 44: 11th NCERT: India Physical Geography


INSIGHTS PRELIMS TEST SERIES 2017

www.insightsonindia.com www.insightsias.com
TEST 8 Solutions

52. Consider the following statements about various writs


mentioned in the Constitution of India.
1. Mandamus is issued when the court finds that a particular office
holder is not doing legal duty and thereby is infringing on the
right of an individual.
2. Certiorari is issued when the executive has prorogued a House
Session before the due date on unreasonable grounds.
Which of the above is/are correct?

a) 1 only
b) 2 only
c) Both 1 and 2
d) None

Solution: a)
Justification: Statement 1: Mandamus means we command. This
writ is a command issued by court to a public official, public body,
corporation, inferior court, tribunal or government asking them to
perform their duties which they have refused to perform.

Due to this, Mandamus is called a wakening call and it awakes the


sleeping authorities to perform their duty. Mandamus thus demands an
activity and sets the authority in action. Mandamus cannot be issued
against the following: a private individual or private body, President,
Governor and a working Chief Justice.

Statement 2: Its a writ that orders to move a suit from an inferior court
to superior court. It is issued by a higher court to a lower court or
tribunal either to transfer a case pending with that to itself or squash its
order. This is generally done because superior court believes that either
the inferior court had no jurisdiction or committed an error of law. Thus,
certiorari is a kind of curative writ.

Q Source: Page 42: Indian Constitution at Work: NCERT XIth


INSIGHTS PRELIMS TEST SERIES 2017

www.insightsonindia.com www.insightsias.com
TEST 8 Solutions

53. Consider the following about the National Human Rights


Commission (NHRC).
1. It can be headed only by the serving Chief Justice of India.
2. It can start an inquiry at its own initiative on a human rights
violation issue.
3. It undertakes and promoting research in the field of human
rights.
4. The commission does not have the power of prosecution.
Select the correct answer using the codes below.

a) 1 and 4 only
b) 2 and 3 only
c) 2, 3 and 4 only
d) 1, 2, 3 and 4 only

Solution: c)
Justification: Statement 1: It is composed of a former chief justice of
the Supreme Court of India, a former judge of the Supreme Court, a
former chief justice of a High Court and two other members who have
knowledge and practical experience in matters relating to human rights.

But, it is not necessary that the Chairman must be the serving CJI, s/he
can be a retired CJI too.

Statement 2 and 3: The commissions functions include inquiry at its


own initiative or on a petition presented to it by a victim into complaint
of violation of human rights; visit to jails to study the condition of the
inmates; undertaking and promoting research in the field of human
rights etc.

Statement 4: The commission does not have the power of prosecution. It


can merely make recommendations to the government or recommend to
the courts to initiate proceedings based on the inquiry that it conducts.

The commissions functions include inquiry at its own initiative or on a


petition presented to it by a victim into complaint of violation of human


INSIGHTS PRELIMS TEST SERIES 2017

www.insightsonindia.com www.insightsias.com
TEST 8 Solutions

rights; visit to jails to study the condition of the inmates; undertaking


and promoting research in the field of human rights etc.

Q Source: Page 43: Indian Constitution at Work: NCERT XIth

54. Consider the following about the United Nations Economic


and Social Commission for Asia and the Pacific (ESCAP).
1. It is the first agency to have its international headquarters in
India.
2. It coordinates the initiatives of major international NGOs
operating in Asia-Pacific.
3. India had acceded to ESCAP membership even before its
independence.
Select the correct answer using the codes below.

a) 1 only
b) 2 and 3 only
c) 3 only
d) 1, 2 and 3

Solution: c)
Justification: Statement 1 and 2: It was established in 1947 with its
headquarters in Bangkok, Thailand, to assist in post-war economic
reconstruction.

ESCAP works to overcome some of the regions greatest challenges by


providing results oriented projects, technical assistance and capacity
building to member States in the following areas:

Macroeconomic Policy, Poverty Reduction and Financing for


Development, Trade and Investment, Transport etc.

In addition, ESCAP gives stronger participation to the smaller and often


left out voices of the region, the least developed countries, the small
island States and landlocked States.


INSIGHTS PRELIMS TEST SERIES 2017

www.insightsonindia.com www.insightsias.com
TEST 8 Solutions

Statement 3: India became its member on 28 March 1947, Pakistan on


30 September 1947. So, 3 is correct.

Q Source: Syllabus: United Nations Website

55. The famous Minerva Mills case was related to


1. Evolution of the basic structure doctrine of the Constitution of
India
2. Status of Directive Principles of State Policy over the
Fundamental Rights
Which of the above is/are correct?

a) 1 only
b) 2 only
c) Both 1 and 2
d) None

Solution: c)
Justification: Statement 1: As per the judgment, Parliament cannot,
under Article 368, expand its amending power so as to acquire for itself
the right to repeal or abrogate the Constitution or to destroy its basic and
essential features. The donee of a limited power cannot be the exercise of
that power convert the limited power into an unlimited one.

Statement 2: The 42nd Amendment had amended Article 31C of the


Constitution to accord precedence to the Directive Principles of State
Policy articulated in Part IV of the Constitution over the Fundamental
Rights of individuals articulated in Part III.

The court held that part of the 42nd Amendment to be unconstitutional


and thus restored the primacy of fundamental rights over the DPSP
except in some situations, which we will cover in later tests.

Q Source: Chapter 3: Indian Constitution at Work: NCERT XIth


INSIGHTS PRELIMS TEST SERIES 2017

www.insightsonindia.com www.insightsias.com
TEST 8 Solutions

56. Which of the following is\are the features of a proportional


representation (PR) system of elections?
1. The entire country can be considered as a single constituency.
2. A Voter can vote for the party rather than the candidate.
3. A party generally gets more seats in proportion to the votes it
receives.
Select the correct answer using the codes below.

a) 1 and 2 only
b) 3 only
c) 1 and 3 only
d) 1, 2 and 3

Solution: a)
Justification: Statement 3: Proportional representation is a type of
electoral system that decides the make-up of a parliament by allocating
seats on the basis of the number of votes each party received. Although
there are many different types of PR, this is the base requirement for a
system to be described as proportional.

Statement 1: Rather than the winner-take all approach of other systems,


PR ensures that votes carry equal weight. To do this, multi-member
constituencies are used. This means that a single area elects more than
one representative. The size of this area can vary according to the
system, ranging from the size of the whole country to a county or local
vicinity.

Statement 2: Voter votes for the party and more than one one
representative representative may be elected from one constituency.

Q Source: Page 59: Indian Constitution at Work: NCERT XIth


INSIGHTS PRELIMS TEST SERIES 2017

www.insightsonindia.com www.insightsias.com
TEST 8 Solutions

57. Consider the following about System of Air Quality and


Weather Forecasting and Research (SAFAR).
1. It was introduced by Union Ministry of Earth Sciences (MoES)
for greater metropolitan cities of India.
2. The main objective of SAFAR project is to increase awareness
among general public regarding the air quality.
Which of the above is/are correct?

a) 1 only
b) 2 only
c) Both 1 and 2
d) None

Solution: c)
Justification: Statement 1: SAFAR was introduced by Union Ministry
of Earth Sciences (MoES) for greater metropolitan cities of India to
provide location specific information on air quality in near real time.

Statement 2: For e.g. aaccording to System of Air Quality and Weather


Forecasting and Research (SAFAR) national capital New Delhis air
quality was recorded in the very poor category. The average (24-hour
rolling) of PM2.5 and PM10, suspended respirable pollutants, were
recorded as unacceptable.

The major reasons for the poor air quality may due to the favourable cold
weather conditions coupled with very light wind, vehicular pollution and
farm fires in neighboring states of Punjab, Haryana.

Q Source: http://www.thehindu.com/todays-paper/delhis-air-quality-
very-poor-safar/article9237392.ece


INSIGHTS PRELIMS TEST SERIES 2017

www.insightsonindia.com www.insightsias.com
TEST 8 Solutions

58. As compared to the Himalayan rivers, the Peninsular rivers


carry very little silt. Why is this the case?
1. Peninsular rivers flow at a much slower rate than the Himalayan
rivers.
2. The Peninsular block is characterized by hard rocky terrain.
Which of the above is/are correct?

a) 1 only
b) 2 only
c) Both 1 and 2
d) None

Solution: b)
Justification: Statement 1: Any rivers that flow at a considerable
speed can erode both lateral banks as well as river bed. Peninsular rivers
generally flow at much faster rate than their Himalayan counterparts. So,
1 is incorrect.

Statement 2: Erosion by a river can take place only when the bank is soft
or river bed is made of erodible soft rocks. Since, peninsular blocks are
hard, rocky, old and consolidated, little silt is accumulated by these
rivers.

Q Source: Page 27: 11th NCERT: India Physical Geography

59. Consider the following statements about a site that is located


in the land of the rising Sun in India.
1. This site is inhabited by the Monpas who have preserved its
Dzong type traditional fortress architecture well.
2. The region reflects Tibeto-Bhutanese as well as North-East
Indian characteristics.
3. It is listed in UNESCO India Tentative List.
The site is?


INSIGHTS PRELIMS TEST SERIES 2017

www.insightsonindia.com www.insightsias.com
TEST 8 Solutions

a) Zurdas
b) Monglozen
c) Nokrek
d) Thembang

Solution: d)
Learning: Thembang, the settlement of the Monpas is a village within
a fortified area measuring approximately 3.2 Acres. It is a host to several
ancient and historical structures and has drawn the maximum attention
for the fortified Dzong constructed using traditional technology of the
region.

The West Kameng district, in which Thembang is located, shares


immediate border with Tibet on the north and Bhutan on the west that
has similar influence on the socio-cultural systems in the district,
including architecture that is manifest in the presence of Dzongs, a type
of fortress architecture that is found in Bhutan and Tibet.

Since Thembang Dzong is believed to be constructed before 1100


CE, it may very well be the precursor to the more majestic Dzong
architecture of Bhutan and a pioneering case study of a fortified
village in the eastern Himalayas. This makes it a very important
site.
The greatest strength of Thembang lies in the traditional
knowledge systems of the Monpas that is indicative of their
consciousness in maintaining the built as well as natural heritage
of their area as their traditional glory.


INSIGHTS PRELIMS TEST SERIES 2017

www.insightsonindia.com www.insightsias.com
TEST 8 Solutions

Q Source: Syllabus: UNESCO Tentative List :


http://www.livemint.com/Politics/T2zS5DucRehRP912lSX7gM/The-
Monpas-Of-Thembang.html

60. WHO has recently reported cases of Rift Valley Fever (RVF)
and Wild Polio in which of these nations?
a) Sierra Leone and Papua New Guinea
b) Mali and South Sudan
c) Syria and Saudi Arabia
d) Nigeria and Niger

Solution: d)
Learning: RVF: In 2016, WHO received reports about unexplained
deaths among humans, along with death and abortion in livestock in the
North Western parts of Niger, and the areas bordering Mali.


INSIGHTS PRELIMS TEST SERIES 2017

www.insightsonindia.com www.insightsias.com
TEST 8 Solutions

Rift Valley fever (RVF) is a viral disease. Those who have bleeding
have a high chance of death. At present, it does not appear that one
person can infect another person.

Its outbreak was reported in 2006/07 in Kenya and Somalia and in


2010 South Africa outbreak.

Poliomyelitis is caused by infection with a member of the genus


Enterovirus known as poliovirus (PV)

In 2014 Polio disease was only spreading between people in


Afghanistan, Nigeria, and Pakistan. In 2015 Nigeria had stopped
the spread of wild poliovirus but it reoccurred in 2016.
Q Source: Current affairs + Syllabus: United Nations:
http://www.who.int/mediacentre/news/releases/2016/nigeria-
polio/en/

61.Consider the following statements.


Assertion (A): The Prime Minister is obliged to furnish all the
information that the President may call for.

Reason (R): Constitutionally, the President has a right to be


informed of all important matters and deliberations of the Council
of Ministers.

In the context of the above, which of these is correct?

a) A is correct, and R is an appropriate explanation of A.


b) A is correct, but R is not an appropriate explanation of A.
c) A is correct, but R is incorrect.
d) Both A and R are incorrect.

Solution: a)
Justification: The President often writes to the Prime Minister and
expresses his views on matters confronting the country. Here, the PM is
bound to share the information that the President has called for.


INSIGHTS PRELIMS TEST SERIES 2017

www.insightsonindia.com www.insightsias.com
TEST 8 Solutions

In doing so, the President exercises his situational discretion.

Learning: Moreover, the President can send back the advice given by
the Council of Ministers and ask the Council to reconsider the decision.

In doing this, the President acts on her own discretion. When the
President thinks that the advice has certain flaws or legal lacunae,
or that it is not in the best interests of the country, the President
can ask the Council to reconsider the decision.
Although, the Council can still send back the same advice and the
President would then be bound by that advice, such a request by
the President to reconsider the decision, would naturally carry a lot
of weight.
Q Source: Page 87: Indian Constitution at Work: NCERT XIth

62. The correct order of these Himalayan ranges from North to


South is?
1. Pir Panjal
2. Zaskar
3. Karakoram
4. Shiwalik
Select the correct answer using the codes below.

a) 3214
b) 2314
c) 2143
d) 1234

Solution: a)
Learning: The Himalayas show a gradual elevation towards the
Dhauldhar and Pir Panjal ranges. Pir Panjal is the largest range of the
lower Himalayas.


INSIGHTS PRELIMS TEST SERIES 2017

www.insightsonindia.com www.insightsias.com
TEST 8 Solutions

Geologically, the Zanskar Range is part of the Tethys Himalaya. It also


separates Kinnaur District from Spiti in Himachal Pradesh. The highest
peaks of Himachal are in the Zanskar Range.

Q Source: Page 12: 11th NCERT: India Physical Geography

63. The President appoints various Ministers and decides their


portfolio on the advice of the
a) Prime Minister
b) Vice-President
c) Speaker, Lok Sabha
d) He makes the decision independently.

Solution: a)
Learning: Formally, a leader who has the support of the majority is
appointed by the President as Prime Minister.

The Prime Minister then decides who will be the ministers in the Council
of Ministers. The Prime Minister allocates ranks and portfolios to the
ministers. Depending upon the seniority and political importance, the


INSIGHTS PRELIMS TEST SERIES 2017

www.insightsonindia.com www.insightsias.com
TEST 8 Solutions

ministers are given the ranks of cabinet minister, minister of State or


deputy minister.

In the same manner, Chief Ministers of the States choose ministers from
their own party or coalition.

Q Source: Page 90: Indian Constitution at Work: NCERT XIth

64. The term Unicorn club often seen in news is used for
a) A club of serial investors who receive funding from international
organizations
b) A group of nations that are promoting traditional modes of
transport such as horse carriage in order to mitigate climate
change
c) Those start-ups that have crossed one billion dollars in
valuation
d) A group of countries that have declared Polo as their national
sport

Solution: c)
Learning: One part of the Indian economy that is witnessing unusual
dynamism is the start-up sector, focused on e-commerce and financial
services.

As of January 2016, there were 19,400 technology-enabled startups in


India, of which 5,000 had been started in 2015 alone.

As of December 2015, eight Indian startups belonged to the Unicorn


club (valuations greater than $1 billion). Flipkart and Ola are some of
these start-ups in India.

Among Indian companies, six of the eight unicorns are consumer-


internet startups.

Q Source: http://timesofindia.indiatimes.com/business/india-
business/Eight-Indian-startups-in-global-1bn-club-
Report/articleshow/47303933.cms


INSIGHTS PRELIMS TEST SERIES 2017

www.insightsonindia.com www.insightsias.com
TEST 8 Solutions

65. Before the 91st Amendment Act (2003), the size of the
Council of Ministers was determined by the
a) Parliament
b) Exigencies of time and situation
c) Total population size of the country
d) Number of constituencies in the country

Solution: b)
Learning: This convention before 2003 led to widespread abuse of the
system.

It led to very large size of the Council of Ministers. Besides, when no


party had a clear majority, there was a temptation to win over the
support of the members of the Parliament by giving them ministerial
positions as there was no restriction on the number of the members of
the Council of Ministers.

This was happening in many States also. Therefore, an amendment was


made that the Council of Ministers shall not exceed 15 percent of total
number of members of the House of People (or Assembly, in the case of
the States).

Q Source: Page 91: Indian Constitution at Work: NCERT XIth

66. The Marrakesh Treaty facilitates


1. Creation and transfer of specially adapted books across national
boundaries for use by visually impaired people
2. Granting permanent Intellectual Property Rights (IPR) to
people with disability across national borders
Which of the above is/are correct?

a) 1 only


INSIGHTS PRELIMS TEST SERIES 2017

www.insightsonindia.com www.insightsias.com
TEST 8 Solutions

b) 2 only
c) Both 1 and 2
d) None

Solution: a)
Justification: Statement 1: The Marrakesh Treaty addresses the book
famine by requiring its contracting parties to adopt national law
provisions that permit the reproduction, distribution and making
available of published works in accessible formats through limitations
and exceptions to the rights of copyright rightholders.

It also provides for the exchange of these accessible format works across
borders by organizations that serve the people who are blind, visually
impaired, and print disabled. It will harmonize limitations and
exceptions so that these organizations can operate across borders.

Statement 2: There is no such provision in the treaty. It is related to the


facilitation of access only.

However, the Treaty is also designed to provide assurances to authors


and publishers (who may not be disabled) that that system will not
expose their published works to misuse or distribution to anyone other
than the intended beneficiaries.

Q Source: Current affairs: Syllabus: United Nations Website

67. Consider the following statements.


1. The United Nations Democracy Fund (UNDEF) finances the
functioning of legislatures in Third World transitioning
Democratic countries.
2. The United Nations Educational, Scientific and Cultural
Organization (UNESCO) is the only UN agency with a mandate
in higher education.
Which of the above is/are correct?

a) 1 only


INSIGHTS PRELIMS TEST SERIES 2017

www.insightsonindia.com www.insightsias.com
TEST 8 Solutions

b) 2 only
c) Both 1 and 2
d) None

Solution: b)
Justification: Statement 1: It was built to support projects that
strengthen the voice of civil society, promote human rights, and
encourage the participation of all groups in democratic processes.

The large majority of UNDEF funds go to local civil society organizations


-- both in the transition and the consolidation phases of democratization.

Statement 2: UNESCO is the only UN agency with a mandate in higher


education. It fosters innovation to meet education and workforce needs.
It examines ways of increasing higher education opportunities for young
people from vulnerable and disadvantaged groups.

Q Source: Syllabus: United Nations Website

68. States that have a bicameral legislature in India are?


1. Jharkhand
2. Karnataka
3. Gujarat
4. Uttar Pradesh
Select the correct answer using the codes below.

a) 1 and 4 only
b) 2 and 3 only
c) 2 and 4 only
d) 1, 2, 3 and 4

Solution: c)
Justification: Seven Indian States, Andhra Pradesh, Telangana, Bihar,
Jammu-Kashmir, Karnataka, Maharashtra and Uttar Pradesh, have


INSIGHTS PRELIMS TEST SERIES 2017

www.insightsonindia.com www.insightsias.com
TEST 8 Solutions

bicameral Legislatures, these are called legislative councils (Vidhan


Parishad).

A bicameral legislature makes it possible to have every decision


reconsidered.

Learning: The Constitution provides for the abolition of the second


chamber in a state where it exists as well as for the creation of such a
chamber in a state where there is none at present.

If a State Legislature passes a resolution by an absolute majority,


together with not less than two-thirds of the members actually present
and voting in favour of the creation of the second chamber and if
Parliament gives concurrence to such a resolution, the concerned State
can have two Houses in the Legislature.

Q Source: Page 103: Indian Constitution at Work: NCERT XIth

69. Consider the following statements.


1. If the Government of India proposes to introduce any new tax, it
must get the approval of the Lok Sabha.
2. The government must give an account about its receipts and
expenditures to the Legislature.
Which of the above is/are correct?

a) 1 only
b) 2 only
c) Both 1 and 2
d) None

Solution: c)
Justification: The Financial powers of the Parliament, involve grant of
resources to the government to implement its programmes.

The government has to give an account to the Legislature about the


money it has spent and resources that it wishes to raise. The legislature


INSIGHTS PRELIMS TEST SERIES 2017

www.insightsonindia.com www.insightsias.com
TEST 8 Solutions

also ensures that the government does not misspend or overspend. This
is done through the budget and annual financial statements.

For e.g. the government must get a money bill passed in the Parliament
to appropriate money from the Consolidated Fund of India.

Q Source: Page : Indian Constitution at Work: NCERT XIth

70. Consider the following statements about Indian temple


architecture.
1. This trend was started by the Chalukyas of Badami.
2. It was essentially a mixture of the nagara and the dravida styles.
3. It was further refined by the Rashtrakutas of Manyakheta.
The above refers to?

a) Panchayatana
b) Vesara
c) Gadag
d) Urushringa

Solution: b)
Learning: The Vesara style contain elements of both Dravida and
Nagara styles.

The Vesara style is also described in some texts as the 'Central Indian
temple architecture style' or 'Deccan architecture'.

The trend was started by the Chalukyas of Badami (500-753AD) who


built temples in a style that was essentially a mixture of the nagara and
the dravida styles, further refined by the Rashtrakutas of Manyakheta
(750-983AD) in Ellora, Chalukyas of Kalyani (983-1195 AD) in
Lakkundi, Dambal, Gadag etc. and epitomized by the Hoysala empire.


INSIGHTS PRELIMS TEST SERIES 2017

www.insightsonindia.com www.insightsias.com
TEST 8 Solutions

Q Source: Revision: 11th NCERT: An Introduction to Indian Arts

71. If the Union Parliament wishes to move a matter from Concurrent


List to the Union List, which of these follows?
a) It must obtain the consent of all State Legislatures for the same.
b) The Rajya Sabha must pass a resolution to this effect approving
the action of the Lok Sabha.
c) The President must consult the Governors of a majority of
States and act according to their advice.
d) The Cabinet must pass an executive fiat to this effect.

Solution: b)
Learning: The Rajya Sabha is an institutional mechanism to provide
representation to the States. Its purpose is to protect the powers of the
States.

In matters of concurrent list, both the Parliament and State Legislatures


can make laws.

Therefore, any matter that affects the States must be referred to Rajya
Sabha for its consent and approval.

Thus, if the Union Parliament wishes to remove a matter from the State
list (over which only the State Legislature can make law) or to either the


INSIGHTS PRELIMS TEST SERIES 2017

www.insightsonindia.com www.insightsias.com
TEST 8 Solutions

Union List or Concurrent List in the interest of the nation, the approval
of the Rajya Sabha is necessary.

The same is true when matter need to be moved from Concurrent List to
the Union list.

This provision adds to the strength of the Rajya Sabha.

Q Source: Page 110: Indian Constitution at Work: NCERT XIth

72. Consider the following about the National Seismic


Programme (NSP).
1. It will trace hydrocarbon resources in sedimentary basins.
2. It will delineate and revise major earthquake prone zones of the
country.
Which of the above is/are correct?

a) 1 only
b) 2 only
c) Both 1 and 2
d) None

Solution: a)
Justification: NSP aims to undertake a fresh appraisal in all
sedimentary basins across India in order to have better understanding of
the hydrocarbon potential of the country.

Under this programme, high-resolution 2D seismic Acquisition,


Processing and Interpretation (API) survey will be conducted across
India especially in the sedimentary basins.

The survey will be conducted by state owned Oil and Natural Gas
Corporation (ONGC) and Oil India Limited (OIL).


INSIGHTS PRELIMS TEST SERIES 2017

www.insightsonindia.com www.insightsias.com
TEST 8 Solutions

The survey project will be completed by 2019. Mahanadi basin; North


eastern states of Assam, Manipur, Arunachal Pradesh, Mizoram and
Nagaland are some of the regions that will be explored.

Q Source: http://economictimes.indiatimes.com/industry/energy/oil-
gas/dharmendra-pradhan-launches-rs-5000cr-national-seismic-
programme-in-mahanadi-basin/articleshow/54816840.cms

73. Recently the tribes Ahom, Koch-Rajbangshi, Moran and


Matak were seen in news for demanding the status of Scheduled
Tribes (SCs). The Central government had also constituted the
Single Committee to examine their demand. These tribes are
natives of
a) Sikkim
b) Assam
c) Arunachal Pradesh
d) Manipur

Solution: b)
Learning: Currently, all of these communities are included in the
OBC/MOBC list.

The agitation to demand Scheduled Tribe (ST) status for 6 communities


in Assam recently erupted again in a bandh due to the delay in the report
of the Singla Committee.

The Committee has consulted representatives of the 6 communities


seeking inclusion in the ST list, as well as those whore opposed to the
demand.

The Ahoms, who belong to the Tai race, ruled Assam for nearly
600 years (1228-1826), and played a major role in uniting the


INSIGHTS PRELIMS TEST SERIES 2017

www.insightsonindia.com www.insightsias.com
TEST 8 Solutions

various ethnic communities of the Brahmaputra valley under a


common Assamese identity.

The Morans and Mataks, whose roots can be traced to the Hukong
Valley in Burma, had migrated to Assam before the Ahoms. They
faced increasing oppression from the second half of the 18th
century; their situation becoming worse after the British annexed
territories in Upper Assam to set up tea gardens.

The Koch-Rajbangshis had their kingdom for several decades in


Assam and present-day North Bengal. The kingdom of Cooch
Behar joined India in August 1949.
Q Source: Current Affairs: Important tribes

74. The National Summit on Fortification of Food was recently


inaugurated in New Delhi. What do you understand by fortification
of food?
a) Increasing the intake of food per capita
b) Nutrient enrichment of food
c) Reducing environmental stress due to food production
d) Development of gated communities around major food centres
to reduce post-harvest losses

Solution: b)
Justification: To address interventions in combating micronutrient
malnutrition in the country, a two day summit was co-hosted by the
Food Safety and Standards Authority of India (FSSAI) in partnership
with related central ministries/departments and development partners.

Food fortification or enrichment is the process of adding


micronutrients i.e. essential trace elements and vitamins into the
food. It is an integrated approach to prevent micronutrient
deficiencies and complements other approaches to improve health
and nutrition.


INSIGHTS PRELIMS TEST SERIES 2017

www.insightsonindia.com www.insightsias.com
TEST 8 Solutions

Food fortification does not require changes in existing food habits


and patterns nor individual compliance. It does not alter
characteristics of food and is socio-culturally acceptable.

It can be introduced quickly and can produce nutritional benefits


and improve health of people in a short period of time. It also safe
and cost effective.

This enrichment process has proven as an effective strategy to


meet the nutritional needs of a large number of people across
various sections of the society, including the poor and
underprivileged, pregnant women and young children.
Learning: The FSSAI has formulated a Food Safety and Standards
(Fortification of Foods) Regulations, 2016, a comprehensive regulation
on fortification of foods in the country.

These regulations set standard guidelines for food fortification and


encourage the production, manufacture, distribution, sale and
consumption of fortified foods.

They also assign specific role of FSSAI in promotion for food fortification
and to make fortification mandatory.

Q Source: http://pib.nic.in/newsite/PrintRelease.aspx?relid=151709

75. Which of these fundamental rights can be found in the South


African Constitution but not in the Indian Constitution?
1. Right to basic and higher education
2. Right to unemployment allowances
3. Rights of religious and cultural minorities
Select the correct answer using the codes below.

a) 1 and 2 only
b) 1 only
c) 2 and 3 only
d) 3 only


INSIGHTS PRELIMS TEST SERIES 2017

www.insightsonindia.com www.insightsias.com
TEST 8 Solutions

Solution: b)
Justification: Rights Common to both the constitutions

Right to fair labour practices in SA constitution is similar to Right


against exploitation and forced labour in Article 23 of the Indian
constitution.
Right of cultural, religious and linguistic communities in SA
constitution is similar to Article 25-30 of the Indian constitution.

Childrens rights in SA constitution are narrowly similar to Right to


Education (Article 21A) in India.
Rights available in South Africa but not in India

A special constitutional court enforces the rights enshrined in the


constitution. It is the SC and HCs in India.

Public interest litigation (PIL) is a unique feature of bill of rights of


SA constitution. So, its a fundamental right to move court for
someone elses violation of fundamental rights. It is neither
implicit nor explicit in Indian constitution. The judiciary has made
it a policy though.

Right to higher education granted in SA is not a Fundamental right


in India, neither is it mentioned in the DPSP.
Q Source: Page 29: Indian Constitution at Work: NCERT XIth

76. Consider the following matches of tribes with the State they
are found in.
1. Nishis : Arunachal Pradesh
2. Angami : Manipur
3. Rengma : Nagaland
4. Adi : Tripura
Select the correct answer using the codes below.


INSIGHTS PRELIMS TEST SERIES 2017

www.insightsonindia.com www.insightsias.com
TEST 8 Solutions

a) 1, 2 and 3 only
b) 1 and 4 only
c) 2, 3 and 4 only
d) 2 and 4 only

Solution: a)
Justification: Nishi Tribe has originated from the Indo-Mongoloid
stock. They are mainly inhabited in the lower region of the Subansiri
district in Arunachal Pradesh. They are considered a large tribe
according to the area and population in Arunachal Pradesh. Their
spoken language is different from the other tribes, which is a part of the
Tibeto-Burman language family.

Angami Tribe community is found in Manipur and has a total population


of 12 million. The tribal community follows Christianity and speak
Tenyidie. They cultivate rice and grains on the hilly areas. The pattern
and design of the clothes worn by the Angami men and women are
almost different from each other.

Rengma Tribe are a Naga tribal community, and are found in both
Nagaland and Assam. The total population is more than 50,000 in entire
Nagaland. The community is further divided into two categories -
Eastern and Western Rengmas. They are considered experts in terrace
cultivation.

Adi Tribe (have two divisions namely Bogums and Bomis) are found in
Arunachal Pradesh. They live on the hills and have their own village
council. This group is again divided into various small sub tribes. Rice
cultivation is practiced by this community and rice serves as the staple
foods for the Adi. Trapping and hunting is also popular.

Q Source: Major tribes North-eastern India

77.Which of the following is correctly matched?


INSIGHTS PRELIMS TEST SERIES 2017

www.insightsonindia.com www.insightsias.com
TEST 8 Solutions

a) Causasus Mountains: Border Baltic Sea


b) Atlantic Highlands : Border Caspian Sea
c) Carpathian mountains: Central Russian Upland
d) Kjolen mountains : Southern Europe

Solution: c)
Learning:

Q Source: Map-based questions: Europe

78. Rainbow revolution is an integral development programme


of agriculture which includes
a) Fisheries
b) Horticulture
c) Forestry


INSIGHTS PRELIMS TEST SERIES 2017

www.insightsonindia.com www.insightsias.com
TEST 8 Solutions

d) All of the above

Solution: d)
Learning: It includes horticulture, forestry, sugarcane, fishery, poultry,
animal husbandry and food processing industries.

https://www.google.co.in/url?sa=t&rct=j&q=&esrc=s&source=we
b&cd=3&cad=rja&uact=8&ved=0ahUKEwiL-
8P50OnPAhUMQ48KHb9DA2oQFggkMAI&url=http%3A%2F%2F
www.thehindu.com%2Fnews%2Fnational%2Fother-
states%2Fagriroadmap-to-pilot-rainbow-revolution-in-bihar-
nitish%2Farticle7522769.ece&usg=AFQjCNG7NB4PYIvW8HHkp
Wu91gzug7_lOQ&sig2=giyS8uZRQJYLANHqaKUxOQ&bvm=bv.1
36499718,d.c2I The various colors of the Rainbow Revolution
indicate various farm practices such as Green Revolution
(Foodgrains), White Revolution (Milk), Yellow Revolution (Oil
seeds), Blue Revolution (Fisheries); Golden Revolution (Fruits);
Silver Revolution (Eggs), Round Revolution (Potato), Pink
Revolution (Meat), Grey Revolution (Fertilizers)

Agriculture is now poised for a Rainbow Revolution fuelled by


technology-induced growth in horticulture, livestock and fisheries
sectors. We will be covering individual sectors in the coming tests,
especially on food processing.
Q Source: http://www.thehindu.com/news/national/other-
states/agriroadmap-to-pilot-rainbow-revolution-in-bihar-
nitish/article7522769.ece

79. When the rivers originate from a hill and flow in all
directions, the drainage pattern is known as radial. This can be
very well seen in the
1. Amarkantak range
2. Mahadeo Hills


INSIGHTS PRELIMS TEST SERIES 2017

www.insightsonindia.com www.insightsias.com
TEST 8 Solutions

Select the correct answer using the codes below.

a) 1 only
b) 2 only
c) Both 1 and 2
d) None

Solution: a)
Justification: Statement 1: This is where the Narmada River, the Son
River and Johila River emerge forming the radial pattern.

Statement 2: The Mahadeo Hills separate the basin of the Narmada


River to the north from that of the Wainganga and Wardha rivers,
tributaries of the Godavari, to the south. The hills have a gentle northern
slope, but drop abruptly to the south onto the Deccan Plateau. They do
not represent such a drainage pattern.

Learning: The drainage pattern resembling the branches of a tree is


known as dendritic the examples of which are the rivers of northern
plain.

When the primary tributaries of rivers flow parallel to each other and
secondary tributaries join them at right angles, the pattern is known as
trellis.

(When the rivers discharge their waters from all directions in a lake or
depression, the pattern is known as centripetal.

Q Source: Chapter 3: India Physical Geography

80. Consider the following with regards to the Geography of


Ukraine.


INSIGHTS PRELIMS TEST SERIES 2017

www.insightsonindia.com www.insightsias.com
TEST 8 Solutions

1. Black Sea guards its coastline.


2. Strait of Gibraltar can be accessed through a narrow passage via
Ukraine.
Which of the above is/are correct?

a) 1 only
b) 2 only
c) Both 1 and 2
d) None

Solution: a)
Justification: Statement 1: Ukraine is currently in territorial dispute
with Russia over the Crimean Peninsula which Russia invaded and
annexed in 2014

It is bordered by Russia to the east and northeast, Belarus to the


northwest, Poland and Slovakia to the west, Hungary, Romania, and
Moldova to the southwest, and the Black Sea and Sea of Azov to the
south and southeast, respectively.

Statement 2: It is a narrow strait that connects the Atlantic Ocean to the


Mediterranean Sea and separates Gibraltar and Peninsular Spain in
Europe from Morocco and Ceuta (Spain) in Africa.

It is far away from Ukraine.


INSIGHTS PRELIMS TEST SERIES 2017

www.insightsonindia.com www.insightsias.com
TEST 8 Solutions

Q Source: Map-based questions: Europe

81. Consider the following statements.


Assertion (A): The Election Commission of India (ECI) can cancel
local body elections in case of rigging and both capturing.

Reason (R): The ECI is responsible for the conduct of local body
elections.

In the context of the above, which of these is correct?

a) A is correct, and R is an appropriate explanation of A.


b) A is correct, but R is not an appropriate explanation of A.
c) A is correct, but R is incorrect.
d) Both A and R are incorrect.


INSIGHTS PRELIMS TEST SERIES 2017

www.insightsonindia.com www.insightsias.com
TEST 8 Solutions

Solution: d)
Justification: Article 324 of the Indian Constitution provides for an
independent Election Commission for the superintendence, direction
and control of the electoral roll and the conduct of elections in India.
These words in the Constitution are very important, for they give the
Election Commission a decisive role in virtually everything to do with
elections.

But, the Election Commission is NOT responsible for the conduct of local
body elections.

The State Election Commissioners work independently of the Election


Commission of India and each has its own sphere of operation.

Only the State Election Commission may take necessary actions for the
successful conduct of these elections.

So, both A and R are wrong.

Q Source: Page 68: Indian Constitution at Work: NCERT XIth

82. Consider the following statements.


1. It was the Mound-Burial system of the Ahom Dynasty.
2. The structural construction and the process of royal burials are
explained in historical documents called Chang-Rung Phukanor
Buranji.
3. The property and Buffer zones are jointly protected and
managed jointly by the Archaeological Survey of India and the
State Department of Archaeology.
The site is?

a) Moidam
b) Burzahom
c) Narcondam


INSIGHTS PRELIMS TEST SERIES 2017

www.insightsonindia.com www.insightsias.com
TEST 8 Solutions

d) Lamjano

Solution: a)
Learning: The Tai-Ahom clan upon their migration from China
established their capital in different parts of the Brahmaputra River
Valley between 12th to 18th CE.

Usurping the Barahi tribe, Chau-lung Siu-ka-pha established the first


capital of the Ahoms at the foothill of Patkai hills and named it Che-rai-
doi or Che-tam-doi, meaning a dazzling city above the mountain in
their language and consecrated site with a ritual.

While the clan moved from city to city, the landscape of Che-Rai-Doi or
Choraideo continued to retain its position as most sacred where the
departed soul of the Royals could transcend into the after-life.

Their unique system of vaulted mounds continued for 600 years, till
many Tai-Ahoms converted to Buddhism while others adopted the
Hindu system of cremation in Moidam, Assam.

There was an effort to conserve the site in 2006, you can read details
here
http://www.telegraphindia.com/1060804/asp/guwahati/story_656802
3.asp

Q Source: Syllabus: UNESCO Tentative List

83. The Loktak Lake has a unique ecosystem called Phumdis


which are actually
a) Water floating mats of soil and vegetation
b) Tall trees that host symbiodinium algae
c) Shola vegetation that extends its deep roots within the lake
d) A symbiotic association of fungi and flagelletes

Solution: a)


INSIGHTS PRELIMS TEST SERIES 2017

www.insightsonindia.com www.insightsias.com
TEST 8 Solutions

Learning: Loktak Lake is the largest natural freshwater lake in the


north-eastern region of India

The largest area of the Phumdi in the Loktak lake is in the Keibul Lamjao
National Park, which is home to Manipur brow-antlered deer also
popularly known as the Sangai.

The habitat exclusively consists of floating meadows and an elevated


strip of hard ground that dissects the park into northern and southern
zones.

For effective in-situ conservation of Sangai, Forest Department of


Manipur in collaboration with Wildlife Institute of India has developed a
plan to reintroduce Sangai in the adjoining area having similar habitat.

The lake is rich in biodiversity and has been designated as a wetland of


International Importance under RAMSAR Convention in 1990.

Many questions on the Loktak lake, phumdis and Sangai have been
asked by UPSC in the past.


INSIGHTS PRELIMS TEST SERIES 2017

www.insightsonindia.com www.insightsias.com
TEST 8 Solutions

Q Source: Syllabus: UNESCO Tentative List

84. Which of the following nations/regions border


Mediterranean Sea?
a) Tunisia, Algeria, Sardinia, Greece and Italy
b) Austria, Hungary, Italy and Spain
c) Poland, Tunisia, Sardinia and Algeria
d) Algeria, Italy, Romania and Morocco

Solution: a)
Learning: Sardinia is the second largest island in the Mediterranean
Sea and an autonomous region of Italy.

Tunisia, Algeria and Morocco are African nations.

Romania, Austria, Poland and Hungary are land-locked states.

Q Source: Map-based questions: Europe


INSIGHTS PRELIMS TEST SERIES 2017

www.insightsonindia.com www.insightsias.com
TEST 8 Solutions

85. Consider the following statements about Protection of Plant


Varieties and Farmers' Rights Authority, India.
1. It was established in pursuance of broad provisions of the Trade
Related Aspects of the Intellectual Property Rights (TRIPS).
2. It grants approval for conducting genetic research and clearing
genetic patents which use biodiversity of India.
Which of the above is/are correct?

a) 1 only
b) 2 only
c) Both 1 and 2
d) None

Solution: a)
Justification: Statement 2: This is done by the National Biodiversity
Authority (NBA) and State Biodiversity authorities.

Statement 1: The authority has been established to provide for the


establishment of an effective system for protection of plant varieties, the
rights of farmers and plant breeders and to encourage the development
of new varieties of plants.

For this purpose, it is necessary to recognize and protect the rights


of the farmers in respect of their contribution made at any time in
conserving, improving and making available plant genetic
resources for the development of the new plant varieties.

Moreover to accelerate agricultural development, it is necessary to


protect plants breeders' rights to stimulate investment for research
and development for the development of new plant varieties.
Such protection facilitates the growth of the seed industry which
will ensure the availability of high quality seeds and planting
material to the farmers.

India having ratified the Agreement on Trade Related Aspects of


the Intellectual Property Rights has to make provision for giving
effect to Agreement.

To give effect to the aforesaid objectives the Protection of Plant


Varieties and Farmers' Rights Act, 2001 has been enacted in India.


INSIGHTS PRELIMS TEST SERIES 2017

www.insightsonindia.com www.insightsias.com
TEST 8 Solutions

For the purposes of this Act, Protection of Plant Varieties and


Farmers' Rights Authority has been established.
http://plantauthority.gov.in/about-authority.htm

Q Source: Improvisation:
http://www.thehindu.com/opinion/letters/patents-and-seed-
industry/article9252480.ece

http://timesofindia.indiatimes.com/city/raipur/CG-serves-2-green-
colour-rice-varieties/articleshow/55011896.cms

86. The quality of government spending increases in which of


these cases?
1. Increasing consumer subsidies
2. More Gross capital formation
3. Higher revenue expenditure
Select the correct answer using the codes below.

a) 1 and 2 only
b) 2 and 3 only
c) 2 only
d) 1 and 3 only

Solution: c)
Justification: Statement 1: Subsidies diverted to wasteful consumption
has a deleterious effects on government finance. For e.g. nearly 38,000
crore rupees were diverted to fertilizer subsidies last year, most of which
found its way in the balance sheet of large companies, illegal exporters
and large farmers. Such subsidies only distort consumption patterns and
do not add to the productive capacity of the economy. High subsidies
reduce the quality of government spending.

Statement 2: Capital formation essential refers to infrastructure. Such


spending increases economic productivity and generates gainful


INSIGHTS PRELIMS TEST SERIES 2017

www.insightsonindia.com www.insightsias.com
TEST 8 Solutions

employment, for e.g. road construction, airport construction, metros etc.


So, 2 is correct.

Statement 3: Revenue expenditure is largely on unproductive activities,


such as staff salaries, pensions, defence expenditure. None of them are
qualitative spending because they do not generate productive long term
economic assets for the economy. So, 3 is incorrect.

Q Source: General debate on quality of government expenditure + Past


year UPSC questions

87. Which of the following bodies/agencies of the United Nations


(UN) has/have won the Nobel Peace Prize?
1. United Nations Childrens Fund (UNICEF)
2. The Office of the United Nations High Commissioner for
Refugees (UNHCR)
3. The Organization for the Prohibition of Chemical Weapons
(OPCW)
4. Intergovernmental Panel on Climate Change (IPCC)
Select the correct answer using the codes below.

a) 1, 2 and 4 only
b) 2 and 3 only
c) 1 and 3 only
d) 1 and 2 only

Solution: a)
Justification: Solve such questions by elimination.

OPCW is not a UN agency, so B and C are incorrect. IPCC works under


the auspices of the UN and has won the Noble Peace Prize in 2007. So,
the only answer can be A.

Statement 1: Reasons can be seen here


http://www.un.org/en/sections/nobel-peace-prize/united-nations-
children%E2%80%99s-fund-unicef/index.html


INSIGHTS PRELIMS TEST SERIES 2017

www.insightsonindia.com www.insightsias.com
TEST 8 Solutions

Statement 3: The OPCW was established to oversee the implementation


of the Chemical Weapons Convention - the full global ban on these kind
of weapons. The Nobel Committee stated: "The conventions and the
work of the OPCW have defined the use of chemical weapons as a taboo
under international law.

Statement 4: It was first established in 1988 by two United Nations


organizations, the World Meteorological Organization (WMO) and the
United Nations Environment Programme (UNEP), and later endorsed by
the United Nations General Assembly.

Q Source: United Nations Website

88. Consider the following about the National Water


Development Agency (NWDA).
1. It is an autonomous agency of the Union Ministry of Water
Resources.
2. It carries out detailed surveys and investigations for possible
river inter-linking sites.
3. It governs the Urban Water Development authorities in
metropolitan areas of India.
Select the correct answer using the codes below.

a) 1 only
b) 1 and 2 only
c) 2 and 3 only
d) 1, 2 and 3

Solution: b)
Justification: Statement 1: It was set up in the year 1982 to carry out
detailed studies, surveys and investigations in respect of Peninsular
Component of National Perspective for Water Resources Development.
The Government subsequently modified the functions of NWDA to
include the Himalayan Component of National Perspective for Water
Resources Development.


INSIGHTS PRELIMS TEST SERIES 2017

www.insightsonindia.com www.insightsias.com
TEST 8 Solutions

Statement 2: It assesses the quantum of water in various Peninsular


River systems and Himalayan River systems which can be
transferred to other basins/States after meeting the reasonable needs of
the basin/States in the foreseeable future.

Statement 3: It is an advisory technical body, not an executive or


administrative body. So, 3 is wrong.

Q Source: http://www.thehindu.com/news/national/kerala/state-says-
no-to-riverlinking/article8762616.ece

89. Which of the following functions are performed by the


Election Commission of India (ECI)?
1. It supervises the preparation of up-to-date voters list.
2. It determines the timing of elections and prepares the election
schedule.
3. It can order a re-poll in a specific constituency on certain
grounds.
4. It accords recognition to political parties and allots symbols to
each of them.
Select the correct answer using the codes below.

a) 1 and 3 only
b) 1 and 2 only
c) 2, 3 and 4 only
d) 1, 2, 3 and 4

Solution: d)
Justification: Statement 1: It makes every effort to ensure that the
voters list is free of errors like nonexistence of names of registered
voters or existence of names of those non-eligible or non-existent.

Statement 2: The election schedule includes the notification of elections,


date from which nominations can be filed, last date for filing
nominations, last date of scrutiny, last date of withdrawal, date of polling
and date of counting and declaration of results.


INSIGHTS PRELIMS TEST SERIES 2017

www.insightsonindia.com www.insightsias.com
TEST 8 Solutions

Statement 3: During this entire process, the Election Commission has


the power to take decisions to ensure a free and fair poll. It can postpone
or cancel the election in the entire country or a specific State or
constituency on the grounds that the atmosphere is vitiated and
therefore, a free and fair election may not be possible.

The Commission also implements a model code of conduct for parties


and candidates. It can order a re-poll in a specific constituency.

It can also order a recount of votes when it feels that the counting
process has not been fully fair and just.

Learning: The Election Commission has very limited staff of its own. It
conducts the elections with the help of the administrative machinery.

However, once the election process has begun, the commission has
control over the administration as far as election related work is
concerned. During the election process, the administrative officers of the
State and central governments are assigned election related duty and in
this respect, the Election Commission has full control over them.

The EC can transfer the officers, or stop their transfers; it can take action
against them for failing to act in a non-partisan manner.

Q Source: Page 71: Indian Constitution at Work: NCERT XIth

90. South-West Monsoon pattern in India is affected by which of


the following?
1. Warm ocean currents over the Indian Ocean
2. Movement of Inter-tropical Convergence Zone (ITCZ)
Which of the above is/are correct?

a) 1 only
b) 2 only
c) Both 1 and 2


INSIGHTS PRELIMS TEST SERIES 2017

www.insightsonindia.com www.insightsias.com
TEST 8 Solutions

d) None

Solution: c)
Justification: Statement 1: These southeast trade winds cross the
equator and enter the Bay of Bengal and the Arabian Sea, only to be
caught up in the air circulation over India. Passing over the equatorial
warm currents, they bring with them moisture in abundance.

If there is an El Nino event, the Ocean currents are not likely to support
Monsoon rainfall in India.

Statement 2: As a result of rapid increase of temperature in May over


the north-western plains, the low pressure conditions over there get
further intensified. ITCZ moves north. By early June, they are powerful
enough to attract the trade winds of Southern Hemisphere coming from
the Indian Ocean.

Q Source: Page 45: 11th NCERT: India Physical Geography

91.The State of Worlds Human Rights report is annually published


by
a) Human Rights Watch (HRW)
b) United Nations Development Programme (UNDP)
c) United Nations Human Rights Council (UNHRC)
d) Amnesty International

Solution: d)
Learning: The Amnesty International Report 2015/16 documents the
state of human rights in 160 countries and territories during 2015.

In many regions, large numbers of refugees were on the move against a


backdrop of conflict and repression, such as in Syria and South Sudan.

Torture and the failure to uphold sexual and reproductive rights were
key concerns.


INSIGHTS PRELIMS TEST SERIES 2017

www.insightsonindia.com www.insightsias.com
TEST 8 Solutions

Government surveillance and impunity continued to deny many their


rights.

This report also celebrates those who stand up for human rights across
the world, often in difficult and dangerous circumstances.

World Report 2015, another report, is published by HRW.

Q Source: International reports : Syllabus

92. You are most likely to come across Lushai and Lepcha tribes
while travelling in
a) Tripura and Sikkim respectively
b) Kohima and Shyok respectively
c) Dehradun and Zaskar respectively
d) Mizoram and Sikkim respectively

Solution: d)
Learning: Lushai tribal community is said to be one of the primitive
tribes of India. Though the Lushai groups are also located in Mizoram,
they are found especially in every hook and nook of the hilly provinces of
Manipur state.

They also have got typical Mongolid features, similar to almost all the
tribes of north-eastern India, especially Manipur.

The Lepchas are the aboriginal inhabitants of Sikkim and they are
mostly settled in North Sikkim. They are mostly Buddhist but
many of them have now adopted Christianity. The lepcha folklores
are melodious and contain lots of interesting folk stories.

Lepchas refer to their language as Rong-aring or Rongring and


falls under the Tibeto-Burman family of languages.
Q Source: Improvisation: Page 15: 11th NCERT: India Physical
Geography


INSIGHTS PRELIMS TEST SERIES 2017

www.insightsonindia.com www.insightsias.com
TEST 8 Solutions

93. In ancient traditions as well as modern times, snakes are


given artistic and reverential representations in many artworks.
Snakes have been traditionally considered as symbols of
a) Source of life
b) Immortality and elixir of life
c) Healing powers and opportunities
d) All of the above

Solution: d)
Learning: In the Hindu regions of Asia the serpent or naga is
considered a nature spirit. It is the protector of springs, wells and rivers.
Serpents bring rain and thus fertility. But, they can also bring floods and
drought.

In many cultures, it is revered as a powerful totem representing the


source of life.

The snake animal meaning is thus powerfully connected to life force and
primal energy.

The Hindus believe that how the naga treats humanity is representative
how we treat the snake and its environment. Serpents are mythically also
believed to carry the elixir of life and immortality.

Q Source: Chapter 9: NCERT XIth: Living Craft Traditions of India

94. Consider the following about Kashmiri Red Stag.


1. It is generally found in dense riverine forests in the high valleys
and mountains.
2. It is protected by Bombay Natural History Society (BNHS) as
their flagship species.
Which of the above is/are correct?

a) 1 only


INSIGHTS PRELIMS TEST SERIES 2017

www.insightsonindia.com www.insightsias.com
TEST 8 Solutions

b) 2 only
c) Both 1 and 2
d) None

Solution: a)
Justification: Statement 1: The Kashmir Stag or Hangul is a
subspecies of elk native to India. Earlier it was believed that it is a
subspecies of red deer. But mitochondrial DNA genetic studies have
revealed that it is part of the Asian clade of elk.

It is found in dense riverine forests in the high valleys and mountains of


Kashmir Valley and northern Chamba district of Himachal Pradesh.

Statement 2: As per Bombay Natural History Society (BNHS) nearly


3000 to 5000 Hanguls existed around the 1940s.

But at present, only about 150 of them survive within its last
bastion in Dachigam National Park located on foothills of
Zabarwan range on the outskirts of Srinagar, J&K.
But, BNHS have not declared it as their flagship species. So, 2 is
wrong.
Learning: It has been listed under Schedule-I of the Wildlife
(Protection) Act, 1972 and J&K Wildlife (Protection) Act, 1978. It also
has been listed among the top 15 species of high conservation priority by
the Central Government. Reasons for decline in population: (i) habitat
destruction, (ii) over-grazing by domestic livestock, and (iii) poaching.

The International Union for Conservation of Nature (IUCN) is going to


declare the Kashmiri Red Stag (also known as Hangul) as a Critically
Endangered species.

Q Source: http://indiatoday.intoday.in/story/kashmir-red-stag-iucn-
critically-endangered/1/787937.html


INSIGHTS PRELIMS TEST SERIES 2017

www.insightsonindia.com www.insightsias.com
TEST 8 Solutions

95. The Kashmir Himalayas are also famous for Karewa


formations, which are useful for the cultivation of
a) Zafran, a local variety of saffron
b) Broccoli and Leek which augment farmers income
c) Cherry tomato and Parsley on the lower reaches
d) Rosemary and Thyme used for ornamental purposes

Solution: a)
Learning: The Karewa sequence, occupying an area of about 2,500 sq
km, rests over the folded Paleozoic-Mesozoic rocks of the Kashmir Basin
in the Kashmir Valley floor, above the river alluvium.

Karewa formations are lake-laid clays and shales. These are lacustine
deposits and appear like flat mounds on the margin of high mountains.

Most of the cultivated fields in the Kashmir Valley are situated on the
Karewa sediments.

Q Source: Page 11: 11th NCERT: India Physical Geography

96. Pitalkhora, Ellora, Nashik and Junnar caves are associated


with which religious sect?
a) Only Hinduism
b) Virashaivas
c) Buddhism
d) Only Alvars

Solution: c)
Learning: The stupas in the fourth and fifth centuries CE have Buddha
images attached.

Junnar has the largest cave excavations more than two hundred caves
around the hills of the townwhereas Kanheri in Mumbai has a hundred
and eight excavated caves.


INSIGHTS PRELIMS TEST SERIES 2017

www.insightsonindia.com www.insightsias.com
TEST 8 Solutions

The most important sites are Ajanta, Pitalkhora, Ellora, Nashik, Bhaja,
Junnar, Karla, Kanheri. Ajanta, Ellora, and Kanheri continue to flourish.

Earlier it was presumed that because of the absence of the Buddha


image, the caves were considered belonging to the orthodox faith of
Buddhism, i.e., the Thervadins, but with the discovery of the Konkan
Maurya inscription mentioning the Saka era 322, i.e., 400 CE, it is now
satisfactorily proved that the cave activity in western Deccan was an
ongoing process.

Q Source: Revision: 11th NCERT: An Introduction to Indian Arts

97. Consider a comparison of the powers of the Lok Sabha and


Rajya Sabha.
1. Both houses can pass a constitutional amendment bill, but the
bill must be first introduced in the Lok Sabha.
2. Both houses approve the proclamation of emergency, but an
emergency cannot be revoked by any of the houses.
Which of the above is/are correct?

a) 1 only
b) 2 only
c) Both 1 and 2
d) None

Solution: d)
Justification: Statement 1: The bill can be introduced in any house.
However it must be passed by both houses. If even one house does not
pass the bill, the bill ends there itself. There is no provision of joint
sitting in such a case.

Statement 2: The proclamation of a war emergency cannot be made by


the president unless the Union cabinet gives him in written that such
proclamation should be made. If a proclamation is NOT revoked


INSIGHTS PRELIMS TEST SERIES 2017

www.insightsonindia.com www.insightsias.com
TEST 8 Solutions

subsequently, it should be laid before the parliament. The both houses of


parliament must approve such proclamation within two months. If the
parliament does not approve the proclamation, it will become ineffective.

It can be revoked either by the President or by Lok Sabha as well.

Q Source: Page 109: Indian Constitution at Work: NCERT XIth

98. Two people A and B claim to own a certain space outside of


earth. A claims a space that is horizontally much wider than that
claimed by B. Which of these observations will be correct in this
context?
a) As space is more likely to have multiple time zones as compared
to that of B.
b) Bs space is more likely to have multiple time zones as compared
to that of A.
c) Depending on the vertical distance between the space regions of
A and B, time difference will vary
d) There will no difference in the time zones in any of the spaces.

Solution: d)
Justification: If a nation is spread on a large geographical area
latitudinally (horizontally on a map), the Sun will be at different
positions in the sky for different latitudes on the nation at the same time.
So, there is a possibility of having multiple time zones. For e.g. in Russia,
there are about 11 time zones.

But, this applies only when the spaces are on earth (which rotates on its
axis). When something is outside of earth, it does not rotate in the same
way as earth does. Moreover, the object need not be so large as earth. So,
the space mentioned in the question will not witness any time zones,
since time zones are essentially created out of earths rotation on its axis,
which is not the case here.


INSIGHTS PRELIMS TEST SERIES 2017

www.insightsonindia.com www.insightsias.com
TEST 8 Solutions

So, D is the correct answer.

Q Source: Improvisation: Chapter 1: 11th NCERT: India Physical


Geography

99. Micro, Small and Medium Enterprises (MSME) play a crucial


role in the Indian economy with a contribution of nearly 40% per
cent to the GDP. Which of the following schemes have been
launched by the government to promote MSMEs in India?
1. Prime Ministers Employment Guarantee Programme (PMEGP)
2. Micro and Small Enterprises Cluster Development Programme
(MSECDP)
3. Credit Guarantee Trust Fund for Micro and Small Enterprises
(CGTMSE)
Select the correct answer using the codes below.

a) 1 and 2 only
b) 2 and 3 only
c) 2 only
d) 1, 2 and 3

Solution: b)
Justification: Statement 1: It is Prime Ministers Employment
Generation Programme (PMEGP). There is no other employment
guarantee programme in India other than MGNREGA.

Statement 2: The Ministry of Micro, Small and Medium Enterprises


(MSME), Government of India (GoI) has adopted the cluster
development approach as a key strategy for enhancing the productivity
and competitiveness as well as capacity building of Micro and Small
Enterprises (MSEs) and their collectives in the country. Clustering of
units also enables providers of various services to them, including banks
and credit agencies, to provide their services more economically, thus
reducing costs and improving the availability of services for these
enterprises.


INSIGHTS PRELIMS TEST SERIES 2017

www.insightsonindia.com www.insightsias.com
TEST 8 Solutions

Statement 3: The Credit Guarantee Fund Scheme for Micro and Small
Enterprises (CGS) was launched by the Government of India (GoI) to
make available collateral-free credit to the micro and small enterprise
sector. Both the existing and the new enterprises are eligible to be
covered under the scheme.

You can read more here http://dcmsme.gov.in/schemes/sccrguarn.htm

Q Source: Economic Survey 2015-16 (we will keep covering some of the
last years sources like this)

100. Consider the following about the Delimitation Commission.


1. It is appointed by the President of India.
2. It works in collaboration with the Election Commission of
India..
3. Its orders cannot be challenged in a court of law.
4. Only the Parliament can modify the orders of the Commission.
Select the correct answer using the codes below.

a) 1 and 2 only
b) 1, 3 and 4 only
c) 2 only
d) 1, 2 and 3 only

Solution: d)
Justification: It is appointed for the purpose of drawing up the
boundaries of constituencies all over the country.

The Delimitation Commission in India is a high power body whose


orders have the force of law and cannot be called in question before
any court.

These orders come into force on a date to be specified by the


President of India in this behalf. The copies of its orders are laid
before the House of the People and the State Legislative Assembly
concerned, but no modifications are permissible therein by them.


INSIGHTS PRELIMS TEST SERIES 2017

www.insightsonindia.com www.insightsias.com
TEST 8 Solutions

Learning: A quota of constituencies to be reserved in each State is fixed


depending on the proportion of SC or ST in that State.

After drawing the boundaries, the Delimitation Commission looks


at the composition of population in each constituency. Those
constituencies that have the highest proportion of Scheduled Tribe
population are reserved for ST.

In the case of Scheduled Castes, the Delimitation Commission


looks at two things. It picks constituencies that have higher
proportion of Scheduled Caste population. But it also spreads these
constituencies in different regions of the State.

This is done because the Scheduled Caste population is generally


spread evenly throughout the country.

These reserved constituencies can be rotated each time the


Delimitation exercise is undertaken.
Q Source: Improvisation: Page 65: Indian Constitution at Work:
NCERT XIth


INSIGHTS PRELIMS TEST SERIES 2017

You might also like